Family Exam #2 questions

Ace your homework & exams now with Quizwiz!

The nurse is assessing a male client who is concerned about his ability to produce enough sperm to have a child. He tells the nurse, "I have had some issues in my younger years." What questions would be important for the nurse to ask this client? Select all that apply. "Do you use drugs or use alcohol excessively? "Are you exposed to X-rays or other radioactive substances?" "Have you ever had any type of trauma or surgery on or near your testicles?" "When was the last time you had sex?" "Do you have a regular girlfriend?"

"Do you use drugs or use alcohol excessively?" "Are you exposed to X-rays or other radioactive substances?" "Have you ever had any type of trauma or surgery on or near your testicles?"

A client who is scheduled for hysteroscopy says to a nurse, 'I thought I would be able to have children, but now I know that will never be possible." Which response would be most appropriate for the nurse to make? "Adoption is always an acceptable option for anyone." "I will call the health care provider to come visit with you." "The use of surrogates is more acceptable today." "Being childless is not as bad as you think it is."

"I will call the health care provider to come visit with you."

The pubescent girl exhibits many changes as a result of estrogen production. Her mother asks the nurse what secondary sexual characteristics to look for in her daughter. What is the best information for the nurse to provide to the mother about secondary sexual characteristics? 1. Smaller stature 2. Lower percentage of body fat tissue 3. Wider hip structure 4. Pubic and axillary hair 5. Breast development 1, 2, 3, 5 1, 2, 4, 5 1, 3, 4, 5 2, 3, 4, 5

1, 3, 4, 5

A nurse is teaching a client who has had a Copper IUD inserted. The nurse will teach the client that the device can stay in place and be effective for up to how long? Fill in the blank with a number.

10 years

A client in her first trimester is concerned about how weight gain will affect her appearance and questions the nurse concerning dietary restrictions. How much weight gain should the nurse point out will be safe for this cleint with a low BMI? 25 to 35 pounds (11 to 16 kilograms) 28 to 40 pounds (13 to 18 kilograms) 15 to 25 pounds (7 to 11 kilograms) 16 to 30 pounds (7.25 to 14 kilograms)

28 to 40 pounds (13 to 18 kilograms)

A married couple, both age 27, is in the reproductive clinic and have been trying for a year to become pregnant. In a year, what percent of couples usually do conceive? 40 percent. 60 percent 90 percent 100 percent

90 percent

A nurse is preparing a client for intrauterine device (IUD) insertion. What should the nurse inform the client when educating her on IUDs? There is reduced menstrual flow after insertion. The insertion procedure is painless. A regular check of threads must be done. There are continuous cramps after insertion.

A regular check of threads must be done.

When conducting a health history with a couple who are experiencing subfertility, the nurse will include which of the following? Select all that apply. General health Medication history Sexual history Surgical history School history

All except school history

A pregnant client is undergoing a fetal biophysical profile. Which parameter of the profile helps measure long-term adequacy of the placental function? Amniotic fluid volume Fetal heart rate Fetal breathing record Fetal reactivity

Amniotic fluid volume

A male client in the hospital receives a visit from his current partner, another male. The client identifies himself as a male and was born with male genitalia. The nurse recognizes that the client could be which of the following sexual orientations? (Select all that apply.) Bisexual Homosexual Transgender Heterosexual

Bisexual Homosexual

A nurse is teaching a female client who is unable to conceive how to monitor her basal body temperature. Which instruction would the nurse prioritize for this client? Record body temperature every night Record menses and time of intercourse Chart body temperature for at least a month Record body weight along with the temperature

Chart body temperature for at least a month

Increased pigmentation on the face of some pregnant women is called: Nigra Melanotropin Striae Chloasma

Chloasma

Many changes occur in the body of a pregnant woman. Some of these are changes in the integumentary system. What is one change in the integumentary system called? Linea rubria Chadwick's sign Ballottement Chloasma

Chloasma

During the initial assessment of a preterm infant, it is noted that the infant has a heart murmur. Which shunt from fetal life most likely remains open? Foramen ovale Ductus arteriosus Umbilical vein Ductus venosus

Ductus arteriosus

When describing genetic disorders to a group of couples planning to have children, the nurse would identify which as an example of an autosomal dominant inheritance disorder? Huntington disease sickle cell disease phenylketonuria cystic fibrosis

Huntington disease

A client reports that her last menstrual period was on August 1. Using Naegele's rule, what would the nurse anticipate as the client's due date? May 8 June 6 May 24 September 15

May 8

The nurse is assessing a client who believes she is pregnant. The nurse points out a more definitive assessment is necessary due to which sign being considered a probable sign of pregnancy? Fatigue Amenorrhea Positive home pregnancy test Nausea and vomiting

Positive home pregnancy test

Blood work results show that a client has a luteal defect, which is suspected to be the primary cause of her subfertility. The nurse explains that this condition is generally corrected by which of the following? Progesterone vaginal suppositories Myomectomy Diathermy Clomiphene citrate

Progesterone vaginal suppositories

A female patient had a tubal ligation 8 years ago. She has remarried, would like to have a child with her current husband, and is requesting a surgical procedure to reopen the tubes. Which information would the nurse most likely include when discussing the success rate for this procedure? The success rate would be zero after this this length of time. The success rate of the operation is not greater than 20% to 30%. The success rate of the operation is not greater than 50% to 60%. The success rate of the operation is not greater than 70% to 80%.

The success rate of the operation is not greater than 70% to 80%

How does fetal circulation differ from circulation after birth? Fetal blood flow bypasses the right atrium and goes directly to the right ventricle. In utero, blood through the pulmonary artery is only 50% of the post-delivery blood flow. The ductus arteriosus carries the majority of the blood circulating from the left atrium to the left ventricle directly to the aorta. The umbilical vein carries oxygenated blood, while deoxygenated blood is carried by the umbilical arteries.

The umbilical vein carries oxygenated blood, while deoxygenated blood is carried by the umbilical arteries.

A client is confused concerning her fertile period each month. The nurse would educate the client by providing which information? The female fertility period is from Day 12 to Day 21 of her menstrual cycle. Fertilization can occur only if intercourse occurs on the day of ovulation. The window of opportunity for conception is from 3 days before to 2 days following ovulation. Since a sperm can only live 12 to 24 hours after ejaculation, fertility is limited to 1 day before to 1 day after ovulation.

The window of opportunity for conception is from 3 days before to 2 days following ovulation

The nurse is explaining about the function of the mons pubis to a client. Which explanation describes the function of the mons pubis? To enhance sexual pleasure To serve as the receptacle for sperm To allow for stretching during childbirth To protect the pelvic bones during sexual intercourse

To protect the pelvic bones during sexual intercourse

What important information should the nurse give a client about the use of a diaphragm during menstruation? Toxic shock syndrome is possible Vaginal discharges may occur It may cause genital burning Use during menstruation can lead to abdominal pain

Toxic shock syndrome is possible

A nurse is counseling women on birth control choices. Which woman is the best candidate for an IUD? a woman who has multiple sex partners a woman who has a history of PID a woman who is in her early twenties and wants children later in life a woman who has one partner and three children

a woman who has one partner and three children

What are methods for delivering hormonal contraception? Select all that apply. orally transdermally vaginally rectally implantation injection

all but rectally

A nurse is teaching a group of patients experiencing secondary subfertility. Which of the following would the nurse include as things that can negatively affect spermatogenesis? (Select all that apply.) Increased scrotal heat Trauma to the testes Drug use Excessive alcohol use X-ray exposure Low body temperature

all except low body temperature

Assessment reveals that a couple is at risk for having a child with a genetic disorder that would arise from the mother. After teaching the couple about reproductive alternatives, the nurse determines that additional discussion is needed when the couple identifies which alternative as being appropriate? alternative insemination by donor surrogate embryo transfer surrogate mother adoption

alternative insemination by donor

Place the following events in the sequence the pregnant woman would experience them, from first to last. All options must be used. uterine elargement quickening amennorhea labor Braxton Hicks contractions

amennorhea uterine enlargement quickening Braxton Hicks contractions labor

After discussing various methods of contraception with a client and her partner, the nurse determines that the teaching was successful when they identify which contraceptive method as providing protection against sexually transmitted infections (STIs)? oral contraceptives tubal ligation condoms intrauterine system

condoms

The nursing instructor is preparing a class presentation covering the various hormones and their functions during pregnancy. The instructor determines the class is successful when the class correctly matches which function with hCG? provides rich blood supply to decidua maintains nutrient-rich decidua continues progesterone production by corpus luteum sustains life of placenta

continues progesterone production by corpus luteum

A community health nurse is preparing a presentation for a health fair on the topics of planning for a pregnancy. Which major goal has the nurse determined should be accomplished with this presentation? Ensure women are using the correct contraceptive method. Ensure couples understand genetic risks. Provide one-on-one counseling. Decrease the number of unwanted pregnancies.

decrease number of unwanted pregnancies

Which assessment finding in the pregnant woman at 12 weeks' gestation should the nurse find most concerning? The inability to: detect fetal heart sounds with a Doppler. feel fetal movements. hear the fetal heartbeat with a stethoscope. palpate the fetal outline.

detect fetal heart sounds with a Doppler.

A woman in the 15th week of gestation is planning on terminating the pregnancy. The procedure done for second trimester terminations is which of the following? menstrual extraction dilatation and curettage dilatation and extraction hysterotomy

dilation and extraction

Which effect would the nurse identify as a normal physiologic change in the renal system due to pregnancy? decrease in glomerular filtration rate dilation of the renal pelvis reduction in kidney size shortening of the ureters

dilation of the renal pelvis

The nurse is prepraing to assess a young woman's external genitalia. The nurse predicts the client is currently in the most fertile phase of the menstrual cycle based on which assessment finding of the vaginal mucus drainage? yellow, tacky, and crumbly distensible, stretchable quality raw egg white consistency clear and oily

distensible, stretchable quality

During a prenatal class for a group of new mothers, the nurse is describing the hormones produced by the placenta. What hormones would the nurse include? Select all that apply. prolactin estriol relaxin progestin human chorionic somatomammotropin

estriol relaxin progestin human chorionic somatomammotropin

A nurse is assessing a woman who has come into the clinic. The nurse obtains a sample of the client's cervical mucus and notes ferning. The nurse interprets this finding to indicate high levels of which hormone? estrogen progesterone oxytocin follicle-stimulating hormone

estrogen

A nurse is providing education related to genetic testing. What would be the benefits that the nurse would explain to the client? Select all that apply. ethnically tied diseases psychosocial disorders inborn errors of metabolism phenotype learning disorders

ethnically tied diseases inborn errors of metabolism

The nurse is teaching a group of college students about the four phases of the sexual response. Which phase will the nurse point out as producing an elevation in heart rate, blood pressure, and respirations? excitement plateau orgasm resolution

excitement

The nurse is developing a presentation for a community group of young adults discussing fetal development and pregnancy. The nurse would identify that the sex of offspring is determined at the time of: meiosis. fertilization. formation of morula. oogenesis.

fertilization.

The nursing student is preparing a presentation which will depict the pre-embryonic stage. The instructor determines the student has correctly illustrated the process by putting the stages in which order? Use all the options. implantation cleavage formation of blastocyst formation of morula formation of zygote

formation of zygote cleavage formation of morula formation of blastocyst implantation

A nurse assesses that a 7-year-old male client has a prominent lower jaw, large ears, speech deficits, and displays hyperactivity. The nurse identifies this as being most likely related to which diagnosis? fragile X syndrome cri-du-chat syndrome Klinefelter syndrome Philadelphia syndrome

fragile X syndrome

A client is admitted in the health care facility with pelvic inflammatory disease (PID). When reviewing the client's history, what would the nurse identify as a risk factor? gestational diabetes frequent douching genetic predisposition environmental exposure

frequent douching

The nurse is creating an educational pamphlet for pregnant mothers. Which is the best description of fetal development for the nurse to emphasize? age in weeks and systems developed length, weight, sex gestational age, length, weight, and systems developed sex and systems developed

gestational age, length, weight, and systems developed

A newborn is diagnosed with ophthalmia neonatorum. The nurse understands that this newborn was exposed to which infection? syphilis Candida albicans gonorrhea human immunodeficiency virus

gonorrhea

A pregnant client tells the nurse that she has a 2-year-old child at home who was born at 38 weeks; she had a miscarriage at 9 weeks; and she gave birth to a set of twins at 34 weeks. Which documentation would be appropriate for the nurse? gravida 2, para 1 gravida 4, para 2 gravida 5, para 4 gravida 5, para 4

gravida 5, para 4

The nurse is caring for a 14-year-old girl who fears she might have a sexually transmitted infection. Which assessment finding would be most indicative of trichomoniasis? green vaginal discharge urinary incontinence flu-like symptoms lesions on the vulva

green vaginal discharge

The nurse is discussing the insulin needs of a primaparous client with diabetes who has been using insulin for the past few years. The nurse informs the client that her insulin needs will increase during pregnancy based on the nurse's understanding that the placenta produces: hCG, which increases maternal glucose levels. hPL, which deceases the effectiveness of insulin. estriol, which interferes with insulin crossing the placenta. relaxin, which decreases the amount of insulin produced.

hPL, which deceases the effectiveness of insulin.

The nurse is assessing the laboratory test results of a client with abnormal uterine bleeding (AUB). Which finding should the nurse prioritize? negative pregnancy test hemoglobin level of 10.1 g/dL prothrombin time of 40 seconds serum cholesterol of 140 mg/dL

hemoglobin level of 10.1 g/dL

A nurse assesses a 32-year-old primigravida client with twin gestation in her second trimester. The client reports constipation from iron supplements. Which condition should the nurse assess for in this client as a result of the constipation? gastric ulcer hemorrhoids thrombophlebitis ptyalism

hemorrhoids

The nurse is preparing the client for the routine laboratory tests which will be obtained at the first prenatal visit. Which test will the nurse prioritize at this visit? prolactin levels hepatitis screen magnesium level rubeola titer

hepatitis screen

A nurse is taking a history during a client's first prenatal visit. Which assessment finding would alert the nurse to the need for further assessment? history of exercising twice a week history of diabetes for 4 years history of occasional use of OTC pain relievers maternal age of 28 years

history of diabetes for 4 years

A nurse is reviewing the medical record of a client who has come to the clinic for an evaluation and reproductive life planning with oral contraceptives. Which information if found on the client's medical record would alert the nurse to a possible contraindication for oral contraceptives? Select all that apply. 31 years of age history of migraine headaches with aura smoking 3 to 5 cigarettes a day history of deep vein thrombosis diabetic retinopathy

history of migraine headaches with aura history of deep vein thrombosis diabetic retinopathy

A young patient pregnant for the second time is scheduling her first prenatal appointment. She has a 5-year-old daughter; her recently widowed mother lives with her, her husband, and her child as well. Whom should the nurse ideally include in this first prenatal visit? husband child boss at work mother patient

husband child mother patient

An anatomy professor is teaching a class of pre-nursing students about the female reproductive system. One student asks, "Where is the opening to the vagina?" The best answer the professor can give is: -between where you pee and where you poop. -in what is called the vestibule, which is located between the labia minora. -by the Bartholin glands. -between the labia majora and by the paraurethral glands.

in what is called the vestible, which is located between the labia minora

A nurse conducting a presentation for a group of nurses explains the changes in various body systems related to pregnancy. The nurse determines that additional teaching is needed when the group chooses which component as contributing to the pregnant woman's hypercoagulable state? increased levels of fibrin increased clotting factors increased plasma fibrinogen increased number of red blood cells

increased number of red blood cells

A 28-year-old primigravida client with diabetes mellitus, in her first trimester, comes to the health care clinic for a routine visit. The client reports frequent episodes of sweating, giddiness, and confusion. What should the nurse tell the client about these experiences? tissue sensitivity to insulin increases as pregnancy advances use of insulin needs to be reduced as pregnancy advances increased secretion of insulin occurs in the first trimester insulin resistance becomes minimal in the latter half of the pregnancy

increased secretion of insulin occurs in the first trimester

The nursing instructor is presenting the basic physiologic changes in the woman which can occur during a pregnancy. The instructor determines the session is successful when the students correctly choose which change in the respiratory function during pregnancy as normal? increased tidal volume increased expiratory volume decreased respiratory rate decreased oxygen consumption

increased tidal volume

A nurse is preparing a presentation for a group of new nurses about the potential for misuse of genetic discoveries and advances. Which aspect would the nurse most likely address? gene replacement therapy for defective genes individual risk profiling and confidentiality greater emphasis on the causes of diseases slower diagnosis of specific diseases

individual risk profiling and confidentiality

During a routine antepartal visit, a pregnant woman says, "I've noticed my gums bleeding a bit since I've become pregnant. Is this normal?" The nurse bases the response on the understanding of which effect of pregnancy? elevated progesterone levels increased venous pressure influence of estrogen and blood vessel proliferation effects of regurgitation from relaxation of the cardiac sphincter

influence of estrogen and blood vessel proliferation

The nurse is preparing to assess a client who is noted to have a history of dysmenorrhea. The nurse predicts the client will report which potential symptom during the examination? intermittent, sharp suprapubic pain abnormally long, heavy menstrual periods dysphoria chronic pelvic pain

intermittent, sharp suprapubic pain

The nurse is preparing to assess a client who is noted to have a history of dysmenorrhea. The nurse predicts the client will report which potential symptom during the examination? intermittent, sharp suprapubic pain abnormally long, heavy menstrual periods dysphoria chronic pelvic pain

intermittent, sharp suprapubic pain

The nurse is providing contraception counseling to a perimenopausal woman who has had negative reactions to oral contraceptives in the past and would like a long-term, nonhormone-based method that has a high rate of success. Neither she nor her husband wants to undergo surgery, however. Which method should the nurse recommend? intrauterine device transdermal patch subdermal progestin implant tubal ligation

intrauterine device

A woman is undergoing testing which provides a picture-like analysis of the number, form, and size of the woman's chromosomes. The nurse identifies this as: karyotyping. genome. genotype. phenotype.

karotyping

A young female patient who is about to go overseas on active duty in the army comes to the clinic and asks for birth control. After discussing the different options with the patient, which of the following would the nurse suggest as best for this patient? COCs packaged with 21 pills COCs packaged with 28 pills morning-after pills low-dose estrogen/progestin pill (Lybrel)

low-dose estrogen/progestin pill (Lybrel)

The school nurse is teaching a group of adolescent females about the menstrual cycle. The nurse determines the session is successful when the group correctly chooses which phase as part of the ovarian cycle? luteal proliferative menstrual secretory

luteal

A nursing instructor is teaching a group of nursing students about the various hormones involved in the female's menstrual cycle. The instructor determines the session is succcessful when the students correctly point out which hormone initiates ovulation? luteinizing hormone progesterone follicle-stimulating hormone estrogen

luteinizing hormone

Which vaccines are contraindicated during pregnancy since they may transmit a viral infection to the fetus? Select all that apply. measles mumps influenza rubella Tdap vaccine (tetanus toxoid, reduced diphtheria toxoid, and acellular pertussis)

measles mumps rubella

A 47-year-old woman is being seen at the local clinic reporting irregular menstrual cycles. The nurse would explain to the client that the irregularity is most likely due to: pregnancy. menopause. hormonal fluctuations. an underlying endocrine problem.

menopause

A woman is confused after finding out the ultrasound results predict a different due date for the birth of her baby. Which factor should the nurse point out is most likely the reason for the miscalculation of the fetal age? an error in math when calculating mistaking implantation bleeding for LMP amount of weight gain of mother in early weeks of pregnancy not seeking prenatal care in the beginning

mistaking implantation bleeding for LMP

The nursing instructor is illustrating the fetal circulation to a group of nursing students. The instructor determines the session is successful after the students correctly choose which main structure is most responsible for providing nutrition and removing waste from the fetus? amniotic fluid umbilical cord placenta mother's blood

mother's blood

A 47-year-old woman with osteoarthritis and hypertension is diagnosed with breast cancer. She tells the nurse that her mother also suffered from osteoarthritis and hypertension and she developed breast cancer at the age of 51 years. The nurse explains that this could be a result of: X-linked inheritance. autosomal recessive inheritance. autosomal dominant inheritance. multifactorial inheritance.

multifactorial inheritance.

About which childhood communicable disease should the nurse question the male client who is seeking information about subfertility? measles rubella chicken pox mumps

mumps

A nurse is educating a pregnant client about physical changes that can occur in pregnancy. Which conditions are associated with physical changes in pregnancy? Select all that apply. persistent cough Kussmaul respirations nasal stuffiness and sinus problems thoracic breathing instead of abdominal breathing swollen and tender gums

nasal stuffiness and sinus problems thoracic breathing instead of abdominal breathing swollen and tender gums

A client is beginning to take a combined oral contraceptive. Which of the following side effects will the nurse caution the client might be expected? Select all that apply. Nausea Weight loss Headache Breast tenderness Frequent urinary tract infections

nausea headache breast tenderness

A group of nursing students is reviewing information about chromosomal abnormalities. The students demonstrate understanding of this information when they identify what as a common mechanism leading to an abnormal chromosome number? nondisjunction deletion duplication translocation

nondisjunction

A woman has just given birth to a healthy term newborn. Upon assessing the umbilical cord, the nurse would identify what findings as normal? Select all that apply. one vein two veins one artery two arteries one ligament two ligaments

one vein two arteries

A female client has the Huntington's disease gene. She and her husband want to have a child but are apprehensive about possibly transmitting the disease to their newborn child. They have strong views against abortion. They would also like to have their "own" child and would consider adopting only as a last resort. Which action would be most appropriate in this situation? undergoing prenatal diagnosis with prenatal choice of continuing pregnancy using donor gametes for conception of a child opting for a preimplantation genetic diagnosis chancing the conception and birth of a child

opting for a preimplantation genetic diagnosis

A woman has just been prescribed clomiphene citrate to stimulate ovulation. Which possible effect should the nurse warn the woman about? overstimulation of the ovary resulting in potential multiple births extensive bleeding during menstruation elevation of her blood glucose level hypertension

overstimulation of the ovary resulting in potential multiple births

A nurse is teaching a class about natural methods of birth control. Which group would the nurse identify as a good candidate for these methods of family planning? Select all that apply.) perimenopausal women postpartal women adolescents young executives college students

perimenospausal women postpartal women

The nurse is preparing to teach a community class to a group of first-time parents. Which information should the nurse include concerning what the pregnant woman's partner may experience as a normal response? feeling distanced from the mother no changes, only the mother has changes during pregnancy physical symptoms similar to the mother desire to be the woman and give birth

physical symptoms similar to the mother

A woman who is pregnant for the first time has arrived to the labor department thinking she was in labor only to be diagnosed with Braxton Hicks contractions and sent home. Prior to leaving ther unit, the woman asks, "How will I know when it is 'true' labor?" Which signs/symptoms should the woman assoicate with true labor? Select all that apply. pain in back that wraps across the abdomen that increases in frequency and intensity leakage of white to yellow discharge from the nipples pink-tinged blood and mucus mixture on underwear sudden gush of clear fluid coming from the vagina urine leakage after coughing or sneezing

pink-tinged blood and mucus mixture on underwear sudden gush of clear fluid coming from the vagina pain in back that wraps across the abdomen that increases in frequency and intensity leakage of white to yellow discharge from the nipples

Assessment of a pregnant woman reveals oligohydramnios. The nurse would be alert for the development of which condition? maternal diabetes placental insufficiency neural tube defects fetal gastrointestinal malformations

placental insufficiency

A nurse is counseling with a couple in the fertility clinic. In discussing the plan of care, which aspect should the nurse anticipate as being most stressful for the couple? Obtaining the necessary fertility testing Tracking daily basal body temperatures Planning intercourse around ovulation Frequent visits to the fertility clinic

planning intercourse around ovulation

A female client complains of irregular menses. On further assessment, the nurse observes that the client is obese and is developing hirsutism. Her diagnosis indicates an ovarian dysfunction with increased levels of testosterone. The nurse interprets these findings to suggest: premature ovarian failure. polycystic ovarian syndrome. hypothyroidism. adrenal hyperplasia.

polycystic ovarian syndrome.

The nurse is preparing to meet with a couple who have requested counseling concerning family planning. The nurse will prioritize which topics for discussion? Ensuring the couple is legally married Ensuring the correct time frame is followed Possible genetic abnormalities Preventing STIs

possible genetic abnormalities

The nurse is preparing to teach a client and her partner about the hormones involved in menstruation. Which hormone will the nurse point out is secreted by the corpus luteum? luteinizing hormone follicle stimulating hormone progesterone androgen

progesterone

The nurse is preparing to teach a client and her partner about the hormones involved in menstruation. Which hormone will the nurse point out is secreted by the corpus luteum? luteinizing hormone follicle stimulating hormone progesterone androgen

progesterone

A client is questioning the nurse about the various options for contraception. When explaining the implantable form, the nurse should point out it contains which form of contraception? progestin estrogen and progestin concentrated spermicide concentrated estrogen

progestin

The nurse is teaching a family planning class at a health fair. Which physiologic change during the plateau stage of sexual response will the nurse point out? lengthening of the vagina labia minor lightens in color retraction of the clitoris scrotal elevation

retraction of the clitoris

A couple who is in for fertility testing ask the nurse what tests are commonly performed to assess fertility. The nurse replies that there are only three primary tests that are used. What are these tests? semen analysis, ovulation monitoring, and tubal patency assessment semen analysis, urinalysis, and ovulation monitoring serologic test for syphilis, semen analysis, and tubal patency assessment pelvic sonogram, ovulation monitoring, and semen analysis

semen analysis, ovulation monitoring, and tubal patency assessment

An adult client currently has low levels of estrogen and progesterone. At this point in the menstrual cycle, the client will likely: shed her uterine lining. ovulate. become pregnant. experience endometrial regeneration.

shed her uterine lining

A young woman says she needs a temporary contraceptive but has a latex allergy. She mentions that she has a papillomavirus infection. Also, she says she is terrible about remembering to take pills. Which method should the nurse recommend? transdermal contraception female condom cervical cap diaphragm

transdermal contraception

T/F: Dilation and curettage is the method of abortion used when the gestational age of a pregnancy is still less than 13 weeks.

true

T/F: dyspareunia is pain during coitus

true

During the health assessment of a pregnant client who is 30 years old, the nurse discusses preventive breast care. Which recommendation would the nurse include? "A breast examination by a health care provider can help detect if there are any structural problems that may affect breastfeeding." "You should perform monthly breast self-examination." "Beginning now, at age 30, you should have annual mammograms." "You should consider being vaccinated against HPV, a virus associated with the development of breast cancer."

"A breast examination by a health care provider can help detect if there are any structural problems that may affect breastfeeding."

A client at 39 weeks' gestation calls the OB triage and questions the nurse concerning a bloody mucous discharge noted in the toilet after an OB office visit several hours earlier. What is the best response from the triage nurse? "It might be nothing. If it happens again call your provider who is on-call." "If the provider did an exam, it might be just normal vaginal secretions, so don't worry about it." "A one time discharge of bloody mucus in the toilet might have been your mucous plug" "Bloody mucus is a sign you are in labor. Please come to the hospital."

"A one time discharge of bloody mucus in the toilet might have been your mucous plug"

A woman is to undergo chorionic villus sampling as part of a risk assessment for genetic disorders. What statement would the nurse include when describing this test to the woman? "A needle will be inserted directly into your fetus's umbilical vessel to collect blood for testing." "A small amount of amniotic fluid will be withdrawn and collected for analysis." "An intravaginal ultrasound measures fluid in the space between the skin and spine." "A small piece of tissue from the fetal placenta will be removed and analyzed."

"A small piece of tissue from the fetal placenta will be removed and analyzed."

Semen analysis has been ordered for the partner of a client who has been unable to become pregnant. What instructions should the nurse provide to the partner? "Collect the sample after two consecutive days of ejaculating." "Keep the sample refrigerated to ensure the sperm survive." "Bring the sample to the lab immediately after you collect it." "If possible, collect the sample first thing in the morning."

"Bring the sample to the lab immediately after you collect it."

An older female pregnant with her first child develops some pain in her legs associated with warmth to touch. Suspecting a blood clot, an ultrasound is presribed and a peripheral venous thrombosis is diagnosed. Which intervention was likely prescribed for this woman? "Take your blood thinner the same time each day." "Buy and wear medical support hose every day." "Take a baby aspirin every morning and evening to prevent further clot formation." "If you sit at a desk, set a timer and get up and walk every 2 to 3 hours."

"Buy and wear medical support hose every day."

The nurse is completing the initial assessment at the prenatal visit of a pregnant client. Which question should the nurse prioritize when completing the review of systems? "Have you ever had a heart attack?" "Do you have a peptic ulcer?" "Have you had any urinary tract infections?" "Have you had any neurologic diseases?"

"Have you had any urinary tract infections?"

Which question would be most appropriate for a nurse to ask a client to assist in establishing a nursing diagnosis of Deficient knowledge related to measure to promote fertility? "How often do you and your partner engage in intercourse?" "How many sexual partners have you had in your life?" "How long have you not been able to get pregnant?" "Have you ever been diagnosed with a STD or STI?"

"How often do you and your partner engage in intercourse?"

The nurse is providing education about barrier contraceptives. Which statement by the patient indicates a need for further education? "I have to wash my cervical caps after each use." "I can leave my diaphragm in for up to 48 hours." "I can insert my female condoms up to 8 hours before intercourse." "Male latex condoms when used correctly are an effective means of contraceptive and very effective at preventing transmission of STDs."

"I can leave my diaphragm in for up to 48 hours"

A female client who has chosen the cervical cap as her method of contraception states that she knows how to use it. Which statement by the client would indicate to the nurse that the client does understand this method of contraception? "I will remove the cap immediately after every intercourse and reinsert it later." "This method will help protect me from getting STIs." "I do not need any follow-up procedures for this method." "I do not have any spermicidal allergies, so this method is OK for me."

"I do not have any spermicidal allergies, so this method is OK for me"

The nurse is providing education regarding fetal kicks/movement to a 22 weeks primigravid in the family planning clinic. Which client statement indicates an understanding of the teaching? "I will need to feel and record that my baby moves every hour." "I will monitor fetal movement at least for 20 minutes every week." "I will document fetal kicks/movement daily and they should be 10 every 2 hours." "I will not be able to feel the fetus kick until 30 weeks gestation."

"I will document fetal kicks/movement daily and they should be 10 every 2 hours."

A nurse is educating a pregnant client about obtaining a blood sample for an alpha-fetoprotein (AFP) level. Which response by the client indicates that the health teaching was successful? "If my AFP level is high, it could mean there is a problem with my baby's spinal cord." "If my AFP level is negative, it means the baby has no birth defects." "If my AFP level is low, then I won't need to follow up." "If there is a need to get my AFP level tested, a blood sample will be obtained around 11 weeks."

"If my AFP level is high, it could mean there is a problem with my baby's spinal cord."

The nurse is teaching a couple about X-linked disorders because they are concerned that they might pass on hemophilia to their children. Which response indicates the need for further teaching? "The father can't be a carrier if he doesn't have hemophilia." "If the father doesn't have it, then his kids won't either." "If the mother is a carrier, her daughter could be one too." "If the mother is a carrier, her sons will have hemophilia."

"If the father doesn't have it, then his kids won't either."

The nurse is counseling a couple who are concerned about vitamin D-resistant rickets, an X-linked dominant disorder, because a member of the father's family has this disorder. Which statement by the couple indicates that more teaching is necessary? "If the father doesn't have the gene, then his daughter won't have rickets." "If the father has the gene, then his son will not have rickets." "If the mother has the gene, then there is a 50% chance her son will have rickets." "If the father has the gene, then his daughter will have rickets."

"If the father doesn't have the gene, then his daughter won't have rickets."

A nurse is conducting a class for nursing students on genetic inheritance and is focusing specifically on X-linked recessive disorders. Which statement by a nursing student would indicate appropriate learning has occurred? "Males cannot be carriers if they do not have the disorder." "If the female is the carrier, the daughter can be too." "If the male does not have it, the children will not either." "If the female is the carrier, the sons may have the disorder."

"If the female is the carrier, the daughter can be too."

A nurse is educating a client on the basal body temperature method as a form of contraception. Which statement by the client indicates and understanding of when she can expect to see a rise in her temperature? "My temperature will increase with the start of my menses." "If I am pregnant, I will have a temperature spike." "There will be an increase in my temperature right before I ovulate." "Immediately following ovulation my temperature will increase."

"Immediately following ovulation my temperature will increase."

The nurse is teaching a couple about the pros and cons of genetic testing. Which statement by the nurse best describes the limits of genetic testing? "Various genetic tests help the primary care provider choose appropriate treatments." "Genetic testing helps couples avoid having children with fatal diseases." "Genetic tests identify people at high risk for preventable conditions." "Some genetic tests can give a probability for developing a disorder."

"Some genetic tests can give a probability for developing a disorder."

A newly married couple asks the nurse which "natural" way of family planning would be most effective in preventing pregnancy. What is the most appropriate response by the nurse? "I believe that abstinence would work best for your life choices." "Coitus interruptus is the easiest method and most reliable." "By checking your cervical mucus you will know when to abstain." "The basal body temperature method is simple and has a low failure rate."

"The basal body temperature method is simple and has a low failure rate."

The nurse is providing education to a male client on vasectomy. What teaching should the nurse include? Select all that apply. "The procedure will stop the production of sperm." "You will be placed under general anesthesia for the procedure." "The vas deferens is cut or cauterized to stop the passage of sperm." "You can resume sexual intercourse in about one week." "There will be a significant amount of pain following the procedure."

"The vas deferens is cut or cauterized to stop the passage of sperm." "You can resume sexual intercourse in about one week."

A client has been given instructions about a scheduled sonohysterosalpingogram. Which statement, if made by the client, should indicate to a nurse that the client has an adequate understanding of the instructions? "I will be put under general anesthesia for the procedure." "They will place a catheter in my uterus and use an ultrasound to see what the problem is." "This is just an ultrasound procedure, like they do for a baby." "They will surgically implant the ovum during the procedure."

"They will place a catheter in my uterus and use an ultrasound to see what the problem is."

The pregnant client at 6 weeks' gestation asks the nurse if an ultrasound will reveal the sex of the fetus yet. What is the best response by the nurse? "We will be able to determine the sex of the baby today with transvaginal ultrasound." "We will have to wait until the baby is 8 weeks' gestation to be able to determine what the sex is." "We will have to wait until the baby is 16 weeks' gestation to determine what the sex is." "We will have to wait until the baby is 20 weeks' gestation to determine the sex of the baby."

"We will have to wait until the baby is 16 weeks' gestation to determine what the sex is."

A 25 year old having a yearly check-up informs the nurse that she has stopped taking birth control pills because she and her husband want to start a family. She states, "I know I will get pregnant right away, because my mother was very fertile." What is the nurse's best response? "The chance of subfertility increases with age, especially after the age of 20." "Women who used oral birth control pills may have difficulty becoming pregnant for several months after discontinuing them." "Based on your history, you probably won't experience a problem getting pregnant." "It seems that everybody is different when it comes to getting pregnant."

"Women who used oral birth control pills may have difficulty becoming pregnant for several months after discontinuing them"

A client asks the nurse questions about female hygiene and wants to know if she should douche to clean her vagina. Which response by the nurse would be most appropriate? "Only douche if the vagina has an odor." "You do not need to douche; the vagina is self-cleaning." "Only douche if you have itching and burning." "If you want, douching is ok."

"You do not need to douche; the vagina is self-cleaning."

A genetic nurse is taking a health history for a 45-year-old male client with Klinefelter syndrome. How will the nurse note this genetic disorder in the documentation? 47XXY 46XY 45XY 46XXY

47XXY

A genetic nurse is taking a health history for a 1-month-old male client with Edwards syndrome. How will the nurse note this genetic disorder in the documentation? 47XX18- 47XY18+ 47XX18+ 47XY18-

47XY18+

The parents are questioning why their newborn was born deaf when there are no other deaf family members. The nurse could explore possible exposure to a teratogenic agent at which stage of the pregnancy? 6 weeks at fertilization 12 weeks 18 weeks

6 weeks

Which of the following changes, with highest priority, should the nurse teach a pregnant patient to report to the health care provider as soon as possible? Vomiting 2 to 3 times a day during first trimester Abdominal pain coming and going during the third trimester Heartburn awakening her at night during the first trimester Frequent urination, every 1 to 2 hours, during the first trimester

Abdominal pain coming and going during the third trimester

At a 12-week visit, the nurse assesses a pregnant woman who is questioning what the baby may look like at this point. Based on the current stage, which structures would the nurse point out to the client? Budlike structures for arms and legs Beating heart and differentiated fingers and toes Audible heartbeat and identifiable sex characteristics Fully shaped lungs and a defined skeletal system

Audible heartbeat and identifiable sex characteristics

A client is undergoing a genetic test involves microscopic examination of metaphase chromosomes. The nurse determines that which of the following is being done? Direct molecular analysis Linkage analysis Cytogenetic analysis Biochemical assays

Cytogenetic analysis

T/F: A transvestite is someone who believes that he or she is of the opposite gender as his or her biological sex.

False

The nurse is caring for a neonate whose mother received no medical care for either of her pregnancies. When assessing the neonate's status, which would indicate a potential A, B, and O incompatibility? Hypothyroidism Hemolytic anemia Electrolyte deficiencies Abnormal bleeding

Hemolytic anemia

The nurse is assessing a man concerned about his ability to father children because he has an "abnormality of the penis." The nurse finds that the urethral opening is on the ventral surface of the penis. How would the nurse document this finding? Epispadias Erectile dysfunction Cryptorchidism Hypospadias

Hypospadias

A client who suspects she is pregnant asks the nurse about the accuracy of home pregnancy tests. The nurse would tell the client that: Home pregnancy tests often give a false positive result. Their reliability is only about 90%. Some of the home pregnancy tests can detect the presence of hCG within one day of the woman's missed period. The test works best on a midday urine sample.

Some of the home pregnancy tests can detect the presence of hCG within one day of the woman's missed period.

In evaluating expected outcomes of a client, what might a nurse consider priority when working with genetic counseling? (Select all that apply.) The couple receives test results timely. The couple states understanding of inheritance. The couple need no further information. The couple is happily married. The couple demonstrates positive coping skills.

The couple receives test results timely. The couple states understanding of inheritance. The couple demonstrates positive coping skills.

The nurse is assessing a pregnant client in her third trimester who is reporting a first-time occurrence of constipation. When asked why this is happening, what is the best response from the nurse? There is not enough fiber in your diet. The intestines are displaced by the growing fetus. This shouldn't be happening. hCG is delaying peristalsis.

The intestines are displaced by the growing fetus.

A patient is to have an amniocentesis with ultrasound. What does the nurse explain to the patient that amniocentesis can determine? Select all that apply. What type of facial features the fetus will have. How much the fetus will weight at birth. Whether the fetal lungs are mature enough to support respiration outside of the womb. For genetic testing. What the eye color of the baby will be.

Whether the fetal lungs are mature enough to support respiration outside of the womb. For genetic testing.

A client prescribed COC has presented for a routine visit. Which finding upon assessment should the nurse prioritize? abdominal pain small amount of breakthrough bleeding light menstrual flow cramping during menses

abdominal pain

A woman with a history of genital lesions caused by herpes simplex virus (HSV) has been admitted to the labor unit at 39 weeks' gestation. She said she was given medicine prophylaxically a few weeks ago to prevent lesions at the time of birth. Which medication is safe to give to a pregnant women with HSV? acyclovir benzathine penicillin chloroquine mefloquine

acyclovir

As part of her physical examination of a pregnant client, the nurse examines the woman's breasts. Which are healthy breast changes that indicate pregnancy? Select all that apply. areolae darken overall breast size increases blue streaking of veins becomes prominent montgomery tubercles become prominent breasts become softer in consistency hard, painless lumps form

areolae darken overall breast size increases blue streaking of veins becomes prominent montgomery tubercles become prominent

A woman is at 20 weeks' gestation. The nurse would expect to find the fundus at which area? just above the symphysis pubis midway between the pubis and umbilicus at the level of the umbilicus midway between the umbilicus and xiphoid process

at the level of the umbilicus

A female client who comes to the clinic for a visit is diagnosed with Turner's syndrome. The nurse would expect the client to experience which of the following? Compromised or absent fertility Development of cystic fibrosis in male offspring Development of respiratory distress syndrome in her infant Maternal tubal problems

compromised or absent fertility

When preparing a class for a group of pregnant women about nicotine use during pregnancy, the nurse describes the major risks associated with nicotine use including: increased risk of spontaneous abortion. decreased birth weight in neonates. increased risk of stillbirth. increased risk of placenta abruptio.

decreased birth weight in neonates.

A sexually active client comes to the clinic requesting some form of birth control but is concerned about the bad side effects she has heard that birth control pills can cause. Which information should the nurse point out about the positive aspects of this type of contraceptive? Select all that apply. decreased incidence of dysmenorrhea decreased acne improved cycle regularity increased menstrual flow decreased acute pelvic inflammatory disease

decreased incidence of dysmenorrhea decreased acne improved cycle regularity decreased acute pelvic inflammatory disease

A school nurse who is teaching a health course at the local high school is presenting information on human development and sexuality. When talking about the role of hormones in sexual development, which hormone does the nurse teach the class is the most important for developing and maintaining the female reproductive organs? estrogen progesterone androgens follicle-stimulating hormone

estrogen

T/F: The majority of pregnancy terminations are done in cases in which the pregnancy threatens the mother's life.

false

A nurse teaching a couple says that when X-linked recessive inheritance is present in a family, the genogram will reveal that: only males in the family have the disorder. the parents of the affected man have the disorder. sons of an affected man are also affected. a history of boys dying at birth for unknown reasons often exists.

only males in the family have the disorder.

The nurse is assessing the latest laboratory results of a pregnant client who is at 17 weeks gestation. The nurse should prepare to teach the client about which possible defects after noting the maternal serum alpha-fetoprotein level is elevated above normal? fetal hypoxia open spinal defects Down syndrome maternal hypertension

open spinal defects

The OB nursing instructor is talking with students about the sexual response cycle. As they are studying the content, they become aware that there are four phases in the sexual response cycle and identify them as which of the following? Select all that apply. Resolution Refraction Excitement Climax Plateau

resolution excitement climax plateau

When educating parents on the incidence of genetically associated childhood cancers, the nurse will discuss which cancers? Select all that apply. retinoblasoma Wilms tumor neuroblastoma osteomyloma acute myelogenous leukemia

retinoblasoma Wilms tumor neuroblastoma

A woman using the cervical mucus ovulation method of fertility awareness reports that her cervical mucus looks like egg whites. The nurse interprets this as which kind of mucus? spinnbarkeit mucus purulent mucus postovulatory mucus normal pre-ovulation mucus

spinnbarkeit mucus (occurs as ovulation approaches)

A pregnant woman is undergoing testing on her first prenatal visit. A blood specimen is obtained for a rapid plasma reagin test. The nurse explains that this test will evaluate for which condition? syphilis genetic disorder Rh factor determination anemia

syphilis

All of the following activities should be avoided in pregnancy EXCEPT Scuba diving Contact sports such as hockey, boxing, and soccer Horseback riding and downhill skiing Taking a bath

taking a bath

A school nurse is talking to an adolescent, who asks her about why she has monthly menstrual cycles. The best explanation that the nurse can offer the adolescent regarding the menstrual phase is to tell her that: her uterus fills up with blood each month and is passed during the menstrual cycle. the uterine lining is being shed due to lowering of hormone levels. her hormones cause her to accumulate blood in the uterus from ovulation. each month, her uterine lining thins out and vessels close to the surface begin to fill with blood.

the uterine lining is being shed due to lowering of hormone levels.

A woman comes to the clinic reporting intense pruritus and a thick curd-like vaginal discharge. On examination, white plaques on observed on the vaginal wall. The nurse suspects which condition? trichomoniasis bacterial vaginosis vulvovaginal candidiasis chlamydia

vulvovaginal candidiasis

A pregnant client has been diagnosed as having a urinary tract infection (UTI). Which of the following instructions regarding control of the infection should the nurse provide the client? Abstain from sexual intercourse Wear cotton underwear Clean from back to front after urination Restrict the intake of salt in the diet

wear cotton underwear

A client with liver disease is seen in the clinic wanting to begin contraception. The nurse recognizes that which type of contraception is best for this client? Oral combination contraception Subdermal hormone implant Intramuscular injections Vaginal estrogen/progestin ring

Vaginal estrogen/progestin ring

The vagina is a hostile environment for sperm. What characteristic of semen protects sperm from the vaginal environment? acidic fluid alkaline fluid presence of testosterone secretions from seminiferous tubules

alkaline fluid

There are many steps in the process of genetic counseling and testing. Put these steps in the correct chronological order from first to last. All options must be used. physical examination of parents explain results of genetic testing assessment of family history amniocentesis nuchal translucency screening support couple in adjusting to diagnosis

assessment of family history physical examination of parents nuchal translucency screening amniocentesis explain results of genetic testing support couple in adjusting to diagnosis

Which type of Mendelian inherited condition results in both genders being affected equally in a vertical pattern? automosomal recessive inheritance X-linked inheritance automosomal dominant inheritance multifactorial genetic inheritance

automosomal dominant inheritance

Gestational diabetes occurs around the 24th week of gestation. When should every woman be screened for gestational diabetes? between 16 and 20 weeks' gestation between 20 and 24 weeks' gestation between 24 and 28 weeks' gestation between 28 and 32 weeks' gestation

between 24 and 28 weeks' gestation

A clinic nurse is talking to an adolescent who informs the nurse that she is sexually active and wants some type of birth control. Which of the following should the nurse recommend to this patient? coitus interruptus cervical mucus method postcoital douching birth control pills

birth control pills

To screen for chromosomal disorders, a 39-year-old woman is scheduled for a circulating cell-free DNA test. What type of test is this? skin biopsy blood specimen collection ultrasound urine specimen collection

blood specimen collection

A nursing instructor is teaching students about genetic disorders and briefly discusses how women can arrange to have a newborn's cord blood frozen and banked. What does the instructor tell students that this blood may be used for in the future? blood transfusions bone marrow transplantation diagnostic tests future problems with bleeding

bone marrow transplantation

The nurse at a health fair is teaching about the various changes of puberty. Which sequence of events will be best for the nurse to present when illustrating pubertal changes in females? menarche, breast budding, appearance of pubic hair Appearance of pubic hair, menarche, breast budding breast budding, appearance of pubic hair, menarche appearance of pubic hair, breast budding, menarche

breast budding, appearance of pubic hair, menarche

A couple asks the nurse what color eyes their baby will have if the mother has brown eyes and the father has blue eyes. What is the best response by the nurse? blue hazel differing brown

brown

A nurse is assessing a 4-month-old with trisomy 21. Which assessment is the priority? cardiac growth and development nutritional genetic

cardiac

A 36-year-old male client whose wife has been unable to conceive is referred for semen analysis. The client prefers to collect the specimen at home. Which instruction should the nurse provide about collecting and transporting the specimen? Carry the semen specimen in an inner pocket Have intercourse before collecting the semen Place the specimen in the refrigerator before taking it to the laboratory Collect the specimen in an lubricated condom

carry the semen specimen in an inner pocket

A 40-year-old client is in her 10th week of pregnancy. So far, her pregnancy appears to be healthy, with no abnormal results from standard diagnostic tests. Because of her age, however, the nurse would anticipate that the client is a candidate for which diagnostic tests? Select all that apply. chorionic villi sampling amniocentesis nuchal translucency screening maternal serum screening

chorionic villi sampling amniocentesis

Which occupation may expose a fetus to environmental hazards? Select all that apply. nurse anesthetist working in a busy oral surgeon's office oncology nurse working in an outpatient chemotherapy unit short-order cook for a busy deli nurse working for a pulmonologist who administers inhalation ribavirin routinely to the client population preschool teacher who has 20 children in the classroom throughout the day

nurse anesthetist working in a busy oral surgeon's office oncology nurse working in an outpatient chemotherapy unit working for a pulmonologist who administers inhalation ribavirin routinely to the client population

A woman just gave birth to a healthy term newborn. Upon assessing the umbilical cord, the nurse would identify which structures as normal? Select all that apply. one vein two veins one artery two arteries one ligament two ligaments

one vein two arteries

After explaining the function of the seminal vesicles to a class, the instructor determines the need for additional teaching when the students identify which action as a function of the seminal vesicles? nourishes the sperm reduces sperm motility aids sperm to reach the ovum prevents sperm destruction by female antibodies

reduces sperm motility

A couple has come to the infertility clinic because they have been unable to get pregnant even though they have been trying for over a year. Diagnostic tests are planned for the woman to ascertain if ovulation is regular and whether her endometrium is adequately supported for implantation. What test would the nurse expect to have ordered for this woman? Serum progesterone Ovulation list Oocyte viability test Urine testosterone

serum progesterone

The nurse is providing prenatal care to a young couple who is pregnant with their first child. In what period of development would the nurse explain to the couple that most congenital defects would occur? the period of the zygote the period of the embryo the period of the fetus all periods are equally vulnerable

the period of the embryo

A couple has chosen fertility awareness as their method of contraception. The nurse explains that the unsafe period for them during the menstrual cycle would be at which time? midway between the normal menstrual cycle three days before and three days after ovulation six days before the onset of menstruation five days after the first day of the menstrual cycle

three days before and three days after ovulation

A nursing instructor is teaching about causes of infertility and identifies a need for further instruction when a student states which of the following? "A genetic abnormality may cause infertility." "An absence of ovulation may cause infertility." "Diet does not play a role in infertility." "Exercise can influence infertility."

"Diet does not play a role in infertility."

The nurse is providing teaching to an adolescent girl on the function of the female reproductive system. Which statement by the client best reflects understanding of the material presented? "I will always count from the end of my last menstrual cycle to determine when I will ovulate." "A vaginal discharge that is thick, scanty and white in color indicates that I am ovulating." "My menstrual cycle lasts about 4 to 6 days each month but varies sometimes, which is normal." "During ovulation, the egg is released directly into my uterus to help with fertilization."

"My menstrual cycle lasts about 4 to 6 days each month but varies sometimes, which is normal."

The nurse has provided information to a client about oral contraceptives. Which statement by the client would indicate a need for further education? "Some oral contraceptives protect against STIs." "Hormonal oral contraceptives reduce the risk of ovarian cancer." "Oral contraceptives need to be taken on a daily basis." "Some hormonal contraceptives do not contain estrogen and rely instead on progestin only."

"some oral contraceptives protect against STIs"

The nurse is teaching a pregnant woman with a prepregnancy body mass index (BMI) of 26 about recommended weight gain. The nurse determines that the teaching was successful when the woman states that she should gain approximately how much during her pregnancy? 35 to 40 pounds (16 to 18 kilograms) 25 to 35 pounds (11 to 16 kilograms) 28 to 40 pounds (13 to 18 kilograms) 15 to 25 pounds (7 to 11 kilograms)

15 to 25 pounds (7 to 11 kilograms)

A urinalysis is done on a client in her third trimester. Which result would be considered abnormal? Trace of glucose 2+ Protein in urine Specific gravity of 1.010 Straw-like color

2+ Protein in urine

A woman at high risk for having a baby with a genetic disorder has chosen to have chorionic villi sampling (CVS) instead of amniocentesis. At which point in her pregnancy is this test performed? 12 weeks 8 to 10 weeks 4 to 6 weeks 16 weeks

8 to 10 weeks

The nurse is reviewing the process of egg maturation and ovulation with a client. What occurs during ovulation of the ovarian cycle that the nurse should include in the teaching session with the client? Under the influence of follicle-stimulating hormone, several follicles begin to ripen, and the ovum with each begins to mature. The empty ruptured cavity in the ovary becomes the corpus luteum and begins to secrete progesterone and estrogen. About day 14, a surge of hormones cause the ovum to burst through the ovary. The uterus is prepared for implantation of an ovum.

About day 14, a surge of hormones cause the ovum to burst through the ovary.

A client at 16 weeks' gestation is scheduled for prenatal testing. Which of the following would the nurse anticipate as the most likely screening test for congenital anomalies based on the current age of this pregnancy? Cardocentesis. Amniocentesis. Nuchal translucency testing. Chorionic villi sampling.

Amniocentesis

During a routine antepartal visit, a pregnant woman reports a white, thick vaginal discharge. What would the nurse do next? Ask the woman if she is having any itching or irritation. Tell the woman that this is entirely normal. Advise the woman about the need to culture the discharge. Check the discharge for evidence of ruptured membranes.

Ask the woman if she is having any itching or irritation.

A client is scheduled for amniocentesis. Which action by the nurse would be most appropriate when preparing the client for the procedure? Select all that apply: Ask the client to void. Instruct the client to drink 1 L of fluid. Ask the client to lie on her left side. Assess fetal heart rate. Insert an I.V. catheter.

Assess fetal heart rate. Ask the client to void.

The nurse is examining a pregnant female in her third trimester and measuring to determine if fetal growth has increased. Where would the nurse place the measuring tape? Just below the pubis symphysis At the uterine isthmus At the level of the corpus At the top of the fundus

At the top of the fundus

A nurse is conducting an assessment on a female client being seen for infertility issues. Which assessment data leads the nurse to believe that anovulation is occurring? Select all that apply. BMI 14.6 No fluctuation on BBT graph Menses started at 14 Unmarried Blood pressure 112/68 mmHg

BMI 14.6 No fluctuation on BBT graph

A client expresses interest in having an intrauterine device (IUD) placed for contraception. Which finding noted in the health history would indicate to the nurse that this would not be an appropriate contraceptive option? Sexually active since 16 Dysmenorrhea G1P0 Bicornate uterus

Bicornate uterus

Vitamin C deficiency can lead to what adverse effects for the gestating mother? Low weight gain Hyperglycemia Blood disorders such as easy bruising Placental abruption

Blood disorders such as easy bruising

A nursing student is preparing an oral presentation about autosomal recessive inheritance. What must occur for an offspring to demonstrate signs and symptoms of the disorder with this type of inheritance? Both parents must be heterozygous carriers. One parent must have the disease. The mother must be a carrier. The father must be affected by the disease.

Both parents must be heterozygous carriers.

What should a patient expect at her first prenatal visit? Choose the best response. Breast examination, cervical length test, biophysical profile, and pap smear Breast examination, speculum examination, bimanual examination of the uterus, and blood work Complete physical examination, cervical length test, and blood work Partial physical examination, biophysical profile, blood work, and pap smear

Breast examination, speculum examination, bimanual examination of the uterus, and blood work

Which interventions would a pregnant client be taught regarding dietary restrictions during pregnancy? Select all that apply. Discard foods that have been left out at room temperature for two hours. Wash raw fruits and vegetables with hot water and a mild soap. Eat only soft cheeses such as feta or brie. Limit beef intake to 10-12 ounces per week. Don't drink raw or unpasteurized milk.

Don't drink raw or unpasteurized milk. Discard foods that have been left out at room temperature for two hours.

A pregnant woman undergoes a triple/quadruple screen at 16 to 18 weeks' gestation. What would the nurse suspect if the woman's level is decreased? Down syndrome sickle-cell anemia cardiac defects respiratory disorders

Down syndrome

A nurse is conducting education classes at the local high school on reproductive life planning. Which would be appropriate for the nurse to implement during the teaching? Select all that apply. Encouragement of abstinence Proper condom application Various religious viewpoints Sexual transmitted infection statistics

Encouragement of abstinence Proper condom application Sexual transmitted infection statistics

The nurse is conducting the 20-week obstetrics assessment on a client who is questioning the nurse about the development of the fetus. Which new occurring developments can the nurse point out to this client? Eyelids are open. Lungs are fully shaped. Eyebrows and scalp hair are present. A developed startle reflex is evident.

Eyebrows and scalp hair are present.

Which laboratory result would be most important for a nurse to monitor for a client who has recently started letrozole? hCG CBC TSH FSH

FSH

The birth attendant has ordered an ultrasound for a pregnant woman to determine the gestational age of the fetus. What other information may be obtained by this test? Select all that apply. Location of the placenta Oxygen supply to fetus Fetal heart rate related to fetal activity Gender of the fetus Fetal neural tube defects Fetal head size

Fetal head size Gender of the fetus Location of the placenta

The nurse is teaching a group of women about the hormone(s) that makes the ova within the ovaries mature each month. Which hormone is responsible for stimulating the ovum? Follicle-stimulating hormone Luteinizing hormone Estrogen Progesterone

Follicle-stimulating hormone

A nursing instructor is explaining the stages of fetal development to a group of nursing students. The instructor determines the session is successful after the students correctly choose which time period as representing the pre-embyonic stage? From fertilization and to the end of the second week . Approximately 2 weeks after fertilization and to the end of the eighth week. Approximately 9 weeks after fertilization and to birth. Approximately 6 weeks after fertilization and to the end of 8 weeks.

From fertilization and to the end of the second week .

A multigravida client is pregnant for the third time. Her previous two pregnancies ended in an abortion in the first and third month of pregnancy. How will the nurse classify her pregnancy history? G3 P0020 G2 P0020 G2 P1020 G3 P0021

G3 P0020

A nurse is explaining the use of an IUD to a female patient interested in obtaining contraception. Which information would the nurse most likely include in the explanation? The intrauterine device (IUD) is an object that is placed by the patient within the uterus to prevent implantation of a fertilized ovum IUDs are small devices made of flexible plastic that provide irreversible birth control IUDs do not prevent fertilization of the egg IUDs seem to affect the way the sperm or egg moves

IUDs seem to affect the way the sperm or egg moves

The client is reporting pain when the nurse begins the insertion of a urinary catheter. What measure can the nurse take to facilitate comfort for this client? Insert the catheter halfway in then rotate it. Identify the urethral meatus between the labia minora, avoiding the clitoris. Cleanse the area with a betadine swab again before proceeding. Tell the client to bear down when they feel the catheter being inserted.

Identify the urethral meatus between the labia minora, avoiding the clitoris.

The nurse is preparing a pregnant client with severe hypertension for an emergent amniocentesis for possible early delivery of the fetus.The nurse will explain to the client that the health care provider is evaluating which parameter? Fetal renal output Fetal alimentary output Maternal blood makeup Level of fetal surfactants

Level of fetal surfactants

A nurse is provding care to a pregnant woman in her first trimester who has come to the clinic for a follow up visit. During the visit, the nurse teaches the woman about some of the changes that she will be experiencing during her pregnancy. Which information would the nurse include when describing changes in the breast? The areolas becomes lighter in color. Montgomery's tubercles become more prominent. Prolactin, an anterior pituitary hormone, stimulates the breasts to grow. Estrogen causes the breasts to feel nodular.

Montgomery's tubercles become more prominent.

Which of a client's cells are most likely to reproduce by meiosis? Ova Breast tissue Endometrium Cervix

Ova

A client is scheduled to start taking tadalafil for erectile dysfunction. A nurse should teach the client to observe for side effects and notify a health care provider immediately if what occurs? Orchitis Nausea Priapism Weight gain

Priapism

What is the main purpose of the chorionic villi? Adhere the blastocyst to the endometrial lining Form the tissues that will become the placenta Produce amniotic fluid Provide an exchange site for the exchange of nutrients and wastes

Provide an exchange site for the exchange of nutrients and wastes

A 23-year-old client who's 27 weeks pregnant arrives at her physician's office with complaints of fever, nausea, vomiting, malaise, unilateral flank pain, and costovertebral angle tenderness. Which condition would the nurse most likely suspect? Asymptomatic bacteriuria Bacterial vaginosis Pyelonephritis Urinary tract infection (UTI)

Pyelonephritis

A nurse is planning care for a client and her husband recently diagnosed with multiple sclerosis and wanting to prevent pregnancy for now. What is the most appropriate nursing diagnosis for this couple? Readiness for enhanced knowledge regarding contraception options Decisional conflict regarding choice of birth control because of health concerns Altered sexuality pattern related to fear of pregnancy Risk for ineffective health maintenance related to lack of knowledge

Readiness for enhanced knowledge regarding contraception options

The nurse is preparing to help a young couple who have been unsuccessful in conceiving for the past 3 months. Which factor should the nurse prioritize for the woman as she begins this process? Basal body temperature pattern Review medication regimen Rubella titer or immunization status Review dietary habits

Rubella titer or immunization status

A client makes an appointment with an obstetrician and assessment reveals positive Hegar and Chadwick signs. What should the nurse teach the client about these results? The client more likely has a gynecologic disorder rather than pregnancy The client is definitively pregnant Pregnancy cannot be confirmed She is probably pregnant, but this must be confirmed by other means

She is probably pregnant, but this must be confirmed by other means

Which of these cardiac variations, if found in the client who is pregnant, should the nurse recognize as a normal finding in pregnancy? Split S1S2 Premature ventricular contractions S4 (atrial gallop) Soft systolic murmur

Soft systolic murmur

The nurse is reviewing the medical record of a female client who has been unable to conceive. The client is scheduled to undergo a procedure that involves use of an oocyte donated by her sister that will be fertilized in the laboratory with sperm from the client's husband. The fertilized oocyte will then be placed in the client's uterus. The nurse identifies this as which type of procedure? Zygote intrafallopian transfer In vitro fertilization Surrogate embryo transfer Gamete intrafallopian transfer

Surrogate embryo transfer

When describing the characteristics of the amniotic fluid to a pregnant woman, which would the nurse include? It is usually an acidic fluid. It is composed primarily of organic substances. The amount gradually fluctuates during pregnancy. It limits fetal movement in utero.

The amount gradually fluctuates during pregnancy.

A mutligravida client is concerned that she may deliver early. When asking the nurse what is the earliest her baby can be delivered and survive, which time frame would the nurse point out? The end of the second trimester The end of the first trimester The end of the third trimester The end of the fourth trimester

The end of the second trimester

A nurse working in an OB clinic meets a female who is 4 to 6 weeks pregnant. Lab results reveal she is positive for syphilis, so she is treated with IM penicillin. Later in the pregnancy, she is retested, and her serum titer results continue to increase. How should the nurse interpret these results? The initial IM penicillin did not treat the syphilis. The female's syphilis is immune to the usual penicillin treatment. The female has been reinfected with syphilis. The serum titer usually remains high for up to a year, so no further treatment is required.

The female has been reinfected with syphilis.

A group of nursing students are preparing a presentation depicting the fetal circulation. The instructor determines the presentation is successful when the students correctly illustrate which route for the ductus arteriosus? The left to right heart atria The aorta to the pulmonary veins The right ventricle to the aorta The pulmonary artery to the aorta

The pulmonary artery to the aorta

A woman has a positive pregnancy test and comes to the nurse with left lower quadrant pain. Bimanual examination reveals a tender mass. Which of the following is suspected? Threatened abortion Appendicitis Ovarian cyst Tubal pregnancy

Tubal pregnancy

A nurse is providing medication teaching related to monophasic combination oral contraceptive (COC) that a client is being given to prevent unwanted pregnancy. Which is the most important intervention for the nurse to include? Use a second form of contraception for the initial seven days on the COC. Begin the pills as soon as they are prescribed. Eliminate the 7 placebo pills to skip your menses. Take the pill at the same time every day and never double the dose.

Use a second form of contraception for the initial seven days on the COC.

The nurse is teaching a client about a vasectomy. The nurse determines the session is successful when the client correctly chooses which fact concerning the vasectomy? Usually done as an office procedure Will no longer ejaculate May notice permanent increased scrotal swelling Testes will no longer produce sperm.

Usually done as an office procedure

The nurse is explaining the phases of the uterine cycle to a client. What action during the proliferative phase of the uterine cycle should the nurse include in the teaching session with the client? While the ovarian follicles are producing increased amounts of estrogen, the endometrium prepares for possible fertilization with pronounced growth. If fertilization does not occur, the corpus luteum degenerates, and hormonal levels fall. Withdrawal of hormones causes the endometrial cells to change, and menstruation begins. An ovum matures and is released.

While the ovarian follicles are producing increased amounts of estrogen, the endometrium prepares for possible fertilization with pronounced growth.

A male client in the hospital receives a visit from his current partner, another male. The client identifies himself as a male and was born with male genitalia. The nurse recognizes that the client could be which of the following sexual orientations? (Select all that apply.) Bisexual Homosexual Transgender Heterosexual

bisexual homosexual

A woman is to receive methotrexate and misoprostol to terminate a first-trimester pregnancy. When preparing the teaching plan for this client, the nurse understands that misoprostol works by: dilating the cervix. acting as a toxin to the trophoblastic tissue. blocking the action of progesterone on the endometrium. causing uterine contractions to expel the uterine contents.

causing uterine contractions to expel the uterine contents

A nurse is providing education to a couple who have just been told their unborn fetus has Down syndrome. The nurse should explain that this is what type of genetic abnormality? duplication translocation deletion inversion

duplication

A newborn is diagnosed with Turner syndrome. When assessing this child, the nurse would most likely assess which finding? edematous hands and feet cat-like cry small head wide-set eyes

edematous hands and feet

The nurse is advising a pregnant woman during her first prenatal visit regarding the frequency of future visits. Which schedule is recommended for prenatal care? once every 3 weeks for the first 28 weeks, then every 2 weeks until 36 weeks, and then weekly until the birth once every 4 weeks for the first 28 weeks, then every 3 weeks until 36 weeks, and then every 2 weeks until the birth once every 4 weeks for the first 28 weeks, then every 2 weeks until 36 weeks, and then weekly until the birth once every 4 weeks for the first 36 weeks, then weekly until the birth

once every 4 weeks for the first 28 weeks, then every 2 weeks until 36 weeks, and then weekly until the birth

A client who is entering her third trimester comes to the prenatal clinic for a follow-up examination. When assessing the breasts, which findings would the nurse expect? Select all that apply. pallor of the areolae prominent veins hyperpigmentation of the nipple warmth increased sensitivity

prominent veins hyperpigmentation of the nipple increased sensitivity

A woman in the third trimester of her first pregnancy expresses fear about the birth canal being wide enough for her to push the baby through it during labor. She is a petite person, and the baby seems so large. She asks the nurse how this will be possible. To help alleviate the client's fears, the nurse should mention the role of the hormone that softens the cervix and collagen in the joints, which allows dilation and enlargement of the birth canal. What is this hormone? relaxin progesterone estrogen human placental lactogen

relaxin

A group of student nurses are comparing the various events which take place during the sexual arousal of both males and females. The students are able to correctly point out which event is one of the underlying physiologic responses after the session? ovulation vasocongestion climacteric myospasm

vasocongestion

The school nurse is presenting a lecture to adolescents to teach them how conception occurs. Which statement by the nurse would accurately describe this process? "Human life begins with the union of two cells: the zygote and the sperm." "At the time of conception, the ovum determines the sex of the baby." "Conception usually occurs when the ovum is in the outer third of the fallopian tube." "The ovum carries the Y chromosome, and the sperm carries an X or Y chromosome."

"Conception usually occurs when the ovum is in the outer third of the fallopian tube."

After delivery, a postpartum mother discusses nutritional needs with her nurse related to breast-feeding. Which statement by the mother demonstrates understanding of the nurse's teaching on the subject? "I will need to eat more meat and whole grains to increase my zinc intake while I am nursing." "My iron intake will be adequate if I increase my intake of milk and vegetables." "My baby will take much of my protein through the breast milk, so I will supplement my diet with fruit and cheeses. "The amount of milk I will produce is dependent upon my calcium intake while I am nursing."

"I will need to eat more meat and whole grains to increase my zinc intake while I am nursing."

After assessing a woman who has come to the clinic, the nurse suspects that the woman is experiencing abnormal uterine bleeding. Which statement by the client would support the nurse's suspicions? "I've been having bleeding off and on that's irregular and sometimes heavy." "I get sharp pain in my lower abdomen usually starting soon after my period comes." "I get really irritable and moody about a week before my period." "My periods have been unusually long and heavy lately with a lot of bleeding."

"I've been having bleeding off and on that's irregular and sometimes heavy."

According to the Family and Medical Leave Act, a woman is guaranteed the right to how many weeks of unpaid, job-protected leave for the birth of a child?

12 weeks

A nurse in an infertility clinic correctly identifies which of the following patients to be at highest risk for not conceiving? 24 year old who practices yoga 3 times a week 23 year old with anorexia who runs 5 miles a day 18 year old who plays volleyball socially 20-year-old vegetarian

23 year old with anorexia who runs 5 miles a day

A nurse is teaching a client about the use of a diaphragm. What is the maximum time the nurse would advise the client that she can keep a diaphragm in place? Fill in the blank with a number.

24 hours

A nurse is discussing menstruation with a female client. The nurse describes a normal amount of blood loss is approximately: 5 to 20 mL 25 to 60 mL 75 to 100 mL 120 to 150 mL

25 to 60 mL

A pregnant client is scheduled to undergo percutaneous umbilical blood sampling. The nurse understands that the rationale for this test would be to identify which disorder? Heart disease Phenylketonuria Anemia Thalassemia

Anemia

A newly married couple is meeting with the nurse to discuss a temporary method of birth control that is both a natural form and does not employ birth control pills/devices, in keeping with their religious beliefs. Which fertility awareness method should the nurse point out will best meet their request to delay conception until they are ready? CycleBeads lactation amenorrhea method vasectomy coitus interruptus

CycleBeads

The nurse is preparing a care plan for a primigravida client and her partner who are excited about her pregnancy and ask lots of questions on various subjects. Which nursing diagnosis should the nurse prioritize for this client and her partner in this care plan? Health-seeking behaviors Fear related to lack of knowledge Risk of injury Deficient knowledge

Deficient knowledge

A male client being evaluated for subfertility has been instructed to provide a sperm sample for analysis. Which instructions would be most appropriate for the nurse to provide? Select all that apply. Put the sample on ice until delivery. Deliver it within 1 hour of collection. You should be abstinent 2 to 4 days prior. Do not use any lubricants with the sample. You can use the withdrawal method.

Deliver it within 1 hour of collection. You should be abstinent 2 to 4 days prior. Do not use any lubricants with the sample.

The nurse is teaching a pregnant client in her last trimester about interventions to help reduce risks of complications during pregnancy. Which of the following are recommended guidelines? Take a daily shower, but avoid a tub bath because this could harm the fetus. If planning to breastfeed, bathe as normal using soap and water on the nipples. During the last month, rest on the left side for at least an hour, morning and afternoon. Sleep on your back to avoid supine hypotension syndrome.

During the last month, rest on the left side for at least an hour, morning and afternoon.

The nurse is assessing a young female who just found out she is pregnant. She is is now reporting vague abdominal discomfort. After noting the client has a history of PID, the nurse predicts the health care provider will give priority to ruling out which situation? Ectopic pregnancy Repeat PID UTI Endometriosis

Ectopic pregnancy

A client arrives to the clinic very excited and reporting a postive home pregnancy test. The nurse cautions that the home pregnancy test is considered a probable sign and will assess the client for which sign to confirm pregnancy? Positive office pregnancy test Fetal movement felt by examiner Hegar sign Chadwick sign

Fetal movement felt by examiner

A woman comes to the prenatal clinic and undergoes a pelvic exam. The doctor notes a softening of the uterine isthmus. The nurse recognized that this finding is known as what sign? Hegar sign Chadwick sign Quickening Goodell sign

Hegar sign

The nursing instructor is illustrating the various phases of the uterine cycle to a group of nursing students. The instructor determines the session is successful when the students correctly choose which action occuring during the secretory phase? While the ovarian follicles are producing increased amounts of estrogen, the endometrium begins to slough off If fertilization does not occur, the corpus luteum degenerates and hormonal levels fall. Withdrawal of hormones causes the endometrial cells to change, and menstruation begins. An ovum matures.

If fertilization does not occur, the corpus luteum degenerates and hormonal levels fall.

A 45-year-old client and her spouse are present in the clinic. Results of fertility testing indicate that the client has damage to her fallopian tubes. Which would be the most appropriate infertility option for this client? Gamete intrafallopian transfer In vitro fertilization Surrogate mother Ovulation stimulation

In vitro fertilization

The nurse is explaining the latest laboratory results to a pregnant client who is in her third trimester. After letting the client know she is anemic, which heme iron-rich foods should the nurse encourage her to add to her diet? Legumes Dairy Grains Meats

Meats

A nurse is preparing a presentation for a health fair on the topic of vasectomy. Which information should the nurse point out in the information? Relatively easy procedure with few complications Regular sperm counts are not essential Procedure is effective immediately Birth control measures are not required

Relatively easy procedure with few complications

A nurse supervisor observes a nurse massage a client's injection site after giving a dose of depot medroxyprogesterone acetate. What is the priority response by the nurse supervisor? Remind the nurse that this injection should absorb slowly. Thank the nurse for excellent client care. Write up an incident report for the medication error. Call the health care provider immediately.

Remind the nurse that this injection should absorb slowly

The OB nursing instructor is talking with students about the sexual response cycle. As they are studying the content, they become aware that there are four phases in the sexual response cycle and identify them as which of the following? Select all that apply. Resolultion Refraction Excitement Climax Plateau

Resolution Excitement Climax Plateau

A nurse is counseling a pregnant client about Maternal Serum Marker Screening. She wants to go ahead with the test to rule out anencephaly. The client understands the risks associated with the test and is worried about what to do if the test shows a positive result even if the child is completely healthy. Which of the following nursing diagnoses would be most appropriate? Risk for Anxiety related to false-positive test results Deficient Knowledge related to Maternal Serum Marker Screening Decisional Conflict related to prenatal diagnostic testing options Risk for Infection related to performance of invasive procedures

Risk for Anxiety related to false-positive test results

Male and female reproductive systems are complementary; for example, male testes and female ovaries; male scrotum and female labia majora; and male glans penis and female clitoris. What part of the female system is homologous to the spermatic cord in the male? Cardinal ligaments Round ligaments Uterosacral ligaments Broad ligament

Round ligaments

In assessing the dietary intake over the last 24 hours of a pregnant client, which food would be most concerning to the nurse? Medium-well steak and a fresh salad. Smoked salmon and bagels 6 oz. of white tuna with crackers Cooked hot dog on a bun with mustard

Smoked salmon and bagels

The nurse is counseling a couple who are concerned that their children might inherit sickle cell disease. Which response from the couple indicates a need for further teaching? "The father cannot pass the disorder onto his son or the mother to her daughter." "The disorder can be passed on to the children only if both parents have the gene." "If both parents have the gene, there is a 25% chance of the children having the disorder." "Even if the children do not get the disease, they can still be carriers of the gene."

"The father cannot pass the disorder onto his son or the mother to her daughter."

A pregnant woman reports hearing about genetic analysis done on skin cells and wants to have it done. The nurse informs her that she is only at 8 weeks and it is too early for this test. When should the nurse tell this patient that this test can be performed? 12th week 15th week 20th week 28th week

15th week

Nondisjunction of a chromosome results in which diagnosis? Huntingon disease Duchenne muscular dystrophy Marfan syndrome Down syndrome

Down syndrome

The nurse is teaching a group of women about the hormone(s) that makes the ova within the ovaries mature each month. Which hormone is responsible for stimulating the ovum? Follicle-stimulating hormone Luteinizing hormone Estrogen Progesterone

Follicle-stimulating hormone

The nurse is assessing a woman who is pregnant for the first time. Which of the following terms applies to this client? Primigravida Primipara Nulligravida Multipara

Primigravida

A woman is going through the luteal phase of her ovarian cycle. What changes are taking place during this period of time? The follicle develops into the corpus luteum. Progesterone levels decrease if fertilization occurs. Luteinizing hormone (LH) levels quickly decrease. Follicle-stimulating hormone (FSH) levels increase.

The follicle develops into the corpus luteum.

The nurse is explaining the phases of the uterine cycle to a client. What action during the proliferative phase of the uterine cycle should the nurse include in the teaching session with the client? While the ovarian follicles are producing increased amounts of estrogen, the endometrium prepares for possible fertilization with pronounced growth. If fertilization does not occur, the corpus luteum degenerates, and hormonal levels fall. Withdrawal of hormones causes the endometrial cells to change, and menstruation begins. An ovum matures and is released.

While the ovarian follicles are producing increased amounts of estrogen, the endometrium prepares for possible fertilization with pronounced growth

A client has consumed a large quanity of fish during pregnancy and expressed concern about the unborn child. What can mercury exposure during pregnancy cause? Brain and neurologic abnormalities in the fetus Stunted fetal growth Fetal malformations Abortion

brain and neurologic abnormalities in the fetus

A client in the clinic requests birth control pills to prevent pregnancy. Upon taking a sexual history, the nurse realizes that the client has multiple partners. Which type of contraception would be the best for the nurse to suggest? birth control pills condoms calendar method natural method

condoms

After teaching a health education class on the female reproductive cycle, the nurse determines that the teaching was effective when the group identifies which phase as belonging to the ovarian cycle? follicular phase proliferative phase secretory phase ischemic phase

folicular phase

Which hormone would the nurse explain as being primarily responsible for maturation of the ovarian follicle? luteinizing hormone gonadotropin-releasing hormone follicle-stimulating hormone estrogen

follicle-stimulating hormone

A student nurse is assigned to care for a woman who is being admitted for delivery of her baby. Where would the nurse expect to find the urethral meatus? Above the clitoris Below the vaginal opening On the true perineum Within the vestibule

within the vestibule

A nurse is educating a prenatal client at her second visit. The client is worried about "blotchy brown spots" on her forehead. The nurse reassures the client about this change by giving which appropriate response? "Avoid sun because it will make the discoloring darker." "Apply over-the-counter bleach cream to the area once a day." "This discoloring could be the start of skin cancer. We can refer you to the primary care provider." "This discoloring could be permanent, and you could use makeup to cover it up."

"Avoid sun because it will make the discoloring darker."

A 40-year-old client is a high profile local politician and exercises on a daily basis. She states to the clinic nurse "We stopped using birth control 7 months ago and I still have not gotten pregnant. Should we be worried?" What is the nurse's best response? "You are young, give it some more time." "Infertility is defined after 1 year of not getting pregnant." "You are very busy, are you sure you are ready to have a baby?" "You should tell the health care provider today during the visit."

"You should tell the health care provider today during the visit."

A pregnant client who is planning to have genetic testing asks the nurse when she should schedule her amniocentesis. What should the nurse tell the client? 10 weeks 24 weeks 30 weeks 16 weeks

16 weeks

A nurse is evaluating a man for subfertility who has undergone a sperm count as part of the evaluation process. Which result would lead the nurse to suspect a problem? 30 million sperm per ejaculation 45 million sperm per milliliter of seminal fluid 60% of sperm are motile 40% of sperm are normal in shape and form

30 million sperm per ejaculation

After completing an assessment, the nurse suspects a woman has confused implantation bleeding with her period. The nurse predicts the woman is at which week in her pregnancy? 4 weeks 3 weeks 2 weeks 1 week

4 weeks

A woman is in her early second trimester of pregnancy. The nurse would instruct the woman to return for a follow-up visit every: 4 weeks. 3 weeks. 2 weeks. 1 week.

4 weeks.

The nursing instructor is leading a discussion on the process of implantation with a group of nursing students. The instructor determines the session is successful after the students correctly choose which explanation of a blastocyst? A ball of about 16 identical cells that forms when the zygote divides The endometrium that is enriched in nutrients in preparation for pregnancy A group of cells that forms what will become the placenta. An inner layer of cells that is separated from an outer layer of cells by a fluid-filled cavity

An inner layer of cells that is separated from an outer layer of cells by a fluid-filled cavity

A young patient comes to the clinic and requests birth control pills to prevent a pregnancy. She tells the nurse that she is worried that she will get mixed up on what days to take the pills. Which would be the best type of pills to prescribe this patient? COCs packaged with 21 pills COCs packaged with 28 pills COCs packaged with 30 pills COCs packaged with 14 pills

COCs packaged with 28 pills

A mother has just given birth to a baby. Assessment reveals that the the color of the baby's irises is abnormal and contains white specks. The nurse would suspect which condition? Fragile X syndrome Down's syndrome Trisomy 13 Turner syndrome

Down's syndrome

The client is having her blood drawn for a Triple or Quad screen. For what does this test screen? (Select all that apply.) Rubella Down's syndrome Neural tube defects Pre-eclampsia Gestational diabetes

Down's syndrome Neural tube defects

A nurse is providing care to a pregnant woman. To promote optimal outcomes, the nurse would engage in which activity? Select all that apply. Individualized assessment Assistance with social coordination Counseling Authoritarian decision making Teaching

Individualized assessment Counseling Teaching

With sexual stimulation the penis becomes erect. What is the physiologic cause of an erection? Penile sinuses fill up with blood. The fascia becomes firm. Veins in the shaft of the penis vasoconstrict. The corpus cavernous vasodilates.

Penile sinuses fill up with blood

The nurse is caring for a client who is at 37 weeks' gestation and has a biophysical profile of 9. Which nursing action is best? Notify the health care provider immediately. Schedule a health care provider appointment for one week. Tell the client to report to the hospital for a nonstress test. Prepare all records as the client will be admitted for a cesarean section.

Schedule a health care provider appointment for one week.

A nurse is assessing a client who has a history of pelvic inflammatory disease (PID). The nurse would be alert for which complication? Select all that apply. ectopic pregnancy pelvic abscess fibrosis inguinal lymphadenopathy

ectopic pregnancy pelvis abscess fibrosis

A nurse working in a clinic that focuses on genetic counseling identifies which component as important when assessing clients for genetic disorders? Select all that apply. Health history Physical examination of family members Laboratory analysis Pattern of inheritance in a family Pelvic examination

everything except pelvic exam

The nursing instructor is teaching a group of nursing students about the menstrual cycle. The instructor determines the session is successful when the students correctly choose which action as responsible for the increased thickness of the endometrium? the level of the FSH the decreasing level of the progesterone the dropping level of LH the increasing level of estrogen

the increasing level of estrogen

When reviewing normal menstruation with an early adolescent, the nurse would teach that during the second half of a typical menstrual cycle, the endometrium of the uterus becomes: thin and transparent, due to progesterone stimulation. thin and transparent, due to follicle-stimulating hormone. thick and purple-hued, due to estrogen stimulation. thick and purple-hued, due to progesterone stimulation.

thick and purple-hued, due to progesterone stimulation.

A group of nursing students are analyzing the fetal circulation. After the session, the students correctly point out which fetal structure contains the highest concentration of oxygen? umbilical artery umbilical vein ductus arteriosus pulmonary vein

umbilical vein

The nursing instructor is preparing to illustrate the various changes between the nonpregnant and pregnant female body. The instructor should point out the blood volume in the pregnant woman can increase by what percentage? 20% to 25% 30% to 35% 40% to 45% 50% to 55%

40-45%

A 35-year-old client has been told to keep her pulse rate below 140 bpm during workouts because she pregnant. This means that the client will be working roughly what percentage of the suggested pulse rate? 60% 65% 70% 75%

75%

A nurse is visiting with a couple who has recently completed subfertility assessments and are awaiting the results. Throughout the conversation the wife intermittently begins to cry. What would be the most appropriate nursing diagnosis for this couple? Hopelessness related to perception of no viable alternative to usual conception Anticipatory grieving related to failure to conceive or sustain a pregnancy Situational low self-esteem related to the apparent inability to conceive Fear related to possible outcome of subfertility studies

Fear related to possible outcome of subfertility studies

During a vaginal exam, the nurse notes that the lower uterine segment is softened. The nurse documents this finding as: Hegar sign. Goodell sign. Chadwick sign. Ortolani sign.

Hegar sign.

A nurse is conducting a teaching program for a group of pregnant women in their first trimester. As part of the program, the nurse is describing how the fetus receives oxygen and nutrients and eliminates waste products. Which area would the nurse include as the location where this exchange takes place? In the intervillous spaces In the ductous venosus Through the fetal membranes On the maternal side of the placenta

In the intervillous spaces

A young male client asks the nurse about circumcision, since he was never circumcised as an infant. Which rationale would be appropriate for exploring circumcision in an adult male? The client is experiencing premature ejaculation. The client has experienced recurring balanoposthitis and phimosis. The foreskin appears to cover the glans of the penis. His scrotum appears uneven with one testicle higher than the other.

The client has experienced recurring balanoposthitis and phimosis

The nursing instructor is illustrating the circulatory flow between the mother and fetus. The instructor determines the session is successful when the class correctly chooses which structure with which route? The one umbilical artery carries oxygen-rich blood to the fetus from the placenta. The two umbilical arteries carry waste products from the placenta to the fetus. The one umbilical vein carries oxygen-rich blood to the fetus from the placenta. The two umbilical veins carry waste products from the fetus to the placenta.

The one umbilical vein carries oxygen-rich blood to the fetus from the placenta.

The nurse is providing care to a woman who has come to the clinic for suggestions about contraception. When determining the recommendation for a client, which factor would be most important for the nurse to integrate into the plan of care? client's partner's desire for pregnancy sexual orientation of the client client's ability to tolerate side effects desire for protection against sexually transmitted infections (STIs)

client's ability to tolerate side effects

The nursing instructor is teaching a class on the menstrual cycle. The instrucor determines the class is successful when the class correctly chooses which hormone that is responsible for initating ovulation? progesterone luteinizing hormone estrogen follicle-stimulating hormone

luteinizing hormone

The nurse is preparing an outline for a class on the physiology of the male sexual response. Which event would the nurse identify as occurring first? sperm emission penile vasodilation psychological release ejaculation

penile vasodilation

A nurse practitioner working in a high-risk pregnancy clinic is mentoring a new graduate nurset. The new nurst asks if there are any ways to diagnose Down syndrome prior to birth other than an amniocentesis. The nurse tells the new nurse that another test that can diagnose the disorder prior to birth. Which test would the nurse practitioner most likely include? blood test ultrasound X-ray Doppler study

ultrasound

The nurse is preparing to teach a birthing class and intends to alert the couples to potential difficulties, ensuring they have as much accurate information as possible. Which area of the uterus will the nurse explain has the highest risk of rupture during labor? cervix corpus uterine isthmus fundus

uterine isthmus

The nurse is creating a diagram that illustrates the components of the male reproductive system. Which structure would the nurse include as a major structure to help ensure fertilization of the ovum? seminal vesicles prostate gland Cowper glands vas deferens

vas deferens

A client who delivered her baby 3 months ago is seen in the clinic and tells the nurse that she and her husband have yet to resume a sexual relationship. The nurse notes that no contraception is currently being used. What is the most appropriate nursing diagnosis for this client? Altered sexual pattern related to fear of pregnancy Spiritual distress related to partner's preference for contraception Decisional conflict related to unintended pregnancy Powerlessness related to failure of chosen contraceptive

Altered sexual pattern related to fear of pregnancy

The nurse is explaining fetal circulation to a pregnant woman and emphasizes the difference in her baby's circulation from the woman's circulation. The nurse determines that the teaching was successful when the woman correctly chooses which reason for the difference? Fetal blood is thicker than that of adults and needs different pathways. Fetal circulation carries rich, oxygenated blood to vital areas first. Fetal blood has a higher concentration of oxygen and circulates more slowly. Fetal heart rates are rapid and circulation time is double that of adults.

Fetal circulation carries rich, oxygenated blood to vital areas first.

A nursing student is preparing an exhibit for a school health fair on the male reproductive system. The student should point out which pituitary hormone as being necessary in the process of sperm production? Follicle-stimulating hormone. Interstitial cell-stimulating hormone. Testosterone. Luteinizing hormone.

Follicle-stimulating hormone.

A nurse is helping a patient prepare for ultrasound examination. Which intervention would be most appropriate for the nurse to implement? Have the patient drink several full glasses of water before the procedure Have the woman void before the procedure Have the father of the child leave the room to avoid x-ray exposure Place a towel under her left buttock

Have the patient drink several full glasses of water before the procedure

A primapara woman has been asking about all the minor body changes she has been experiencing over the first 3 months like nausea, fatigue, breast tenderness, constipation, and abdominal discomfort. The clinic nurse responsible for the documentation related to this woman's reports has identified which diagnosis to be the most relevant in this scenario? Ineffective coping Health-seeking behaviors Readiness for enhanced comfort Acute distress

Health-seeking behaviors

A pregnant client is visiting the clinic and complains about the tiny, blanched, slightly raised end arterioles on her face, neck, arms, and chest. The nurse should explain that these are normal during pregnancy and referred to as which of the following? Epulis Linea nigra Striae gravidarum Telangiectasias

Telangiectasias

A nurse calls the mother of a 1-month-old infant to conduct preappointment education for genetic screening. What item would the nurse suggest that the client bring to the appointment that will aid in the genetic assessment? insurance card immunization record family photos birth history

family photos

After teaching a health education class on the female reproductive cycle, the nurse determines that the teaching was effective when the group identifies which phase as belonging to the ovarian cycle? follicular phase proliferative phase secretory phase ischemic phase

follicular phase

A couple informs the nurse that they have not been able to conceive for the last year. During assessment, the nurse learns that the couple has a 3-year-old daughter and 7-year-old son. They report having had no problems getting pregnant the other times. Which type of fertility is this couple exhibiting? primary subfertility secondary subfertility fertility sterility

secondary subfertility

A primigravida who is 40 weeks' pregnant thinks she may be in labor. She calls the nurse and reports that she has had 10 contractions in the last hour. She says that they are mildly painful and last 45 seconds. What is the best response from the nurse? "Did your water break? Do you feel the baby moving?" "You should wait 1 hour, and then come to the hospital." "You need to come in right away because you live 20 minutes away." "Wait until your water breaks, and then come into the hospital." "Wait 1 hour and see how the contractions change, and then call back."

"Did your water break? Do you feel the baby moving?"

When educating parents on recessive genetic disease statistics, the nurse understands that which statement by the parents indicates an accurate understanding of genetic inheritance? "My children will have a 50/50 chance of having a genetic disease." "If I have four children, only one will have the disease." "With each child the odds of inheritance decrease significantly." "Each child will have a 25% chance of developing the disease."

"Each child will have a 25% chance of developing the disease."

A client 6 weeks postpartum is preparing to receive a depot medroxyprogesterone acetate injection. Which statement by the client indicates that further teaching is necessary? "I will need to return every 3 months for a follow up." "I need to be patient as it will take a while before I can get pregnant." "By increasing my activity and watching my diet I will better control my weight." "This does not affect my milk supply so I can continue to breast-feed."

"I need to be patient as it will take a while before I can get pregnant."

Which statement made by a client with a chlamydial infection indicates understanding of the potential complications? "I'm glad I'm not pregnant; I'd hate to have a malformed baby from this disease." "I hope this medicine works before this disease gets into my urine and destroys my kidneys." "If I had known a diaphragm would put me at risk for this, I would have taken birth control pills." "I need to treat this infection so it doesn't spread into my pelvis because I want to have children some day."

"I need to treat this infection so it doesn't spread into my pelvis because I want to have children some day."

The nurse is teaching the pregnant woman about nutrition for herself and her baby. Which statement by the woman indicates that the teaching was effective? "I can eat any seafood that I like because it contains phosphorus, which is a nutrient that pregnant women need." "I will need to take iron supplementation throughout my pregnancy even if I am not anemic." "Milk production requires higher levels of calcium; therefore, if I am going to breastfeed, I must take a calcium supplement during pregnancy." "Because I am pregnant, I can eat anything I want and not worry about weight gain."

"I will need to take iron supplementation throughout my pregnancy even if I am not anemic."

A pregnant client is undergoing an amniocentesis in her third trimester and is worried why she is undergoing this procedure. Which statement by the nurse would best alleviate this client's anxieties? "This procedure is not very dangerous and you should not worry about it." "We are concerned that your baby may have some genetic disorder." "The doctor can explain everything to you when we are done with the procedure." "We want to be sure your baby's lungs are mature and this measures the development of the baby's lungs.

"We want to be sure your baby's lungs are mature and this measures the development of the baby's lungs.

A nurse is teaching a group of primigravida woman who are in their first trimester. One of the women asks the nurse about sexual activity during pregnancy. Which information would the nurse most likely incorporate into the response? Female orgasm on the EDC will cause labor to begin. Intercourse is not recommended before 36 weeks because it can induce labor. Women who have a partially dilated or effaced cervix at term must refrain from sexual activity. Because of pelvic congestion, women may experience increased clitoral sensitivity.

Because of pelvic congestion, women may experience increased clitoral sensitivity.

All of the following are premonitory signs of labor EXCEPT Lightening Effacement Bloody show Nesting Chadwick's sign

Chadwick's sign

The nurse is discussing permanent birth control with a female patient and her husband. Which of the following is considered a permanent nonsurgical sterilization methods? Tubal ligation Tubal occlusion Vasectomy Essure

Essure

The nurse is performing an assessment of a woman who has come to a health care facility for a diagnosis of pregnancy. The women is positive for breast changes, nausea, and amenorrhea. On physical exam, it is noted that the client has softening of the cervix. How should the nurse document this in her notes? ballottement Chadwick sign Goodell sign Hegar sign

Goodell sign

A client at 16 weeks' gestation comes to the office for a routine exam. At what location within the abdomen would the nurse anticipate the uterus to be found? At the level of the umbilicus At the xiphoid process Halfway between the symphysis pubis and the umbilicus Below the symphysis pubis

Halfway between the symphysis pubis and the umbilicus

What can the nurse educate a subfertile couple about to avoid the problem with lower sperm counts in the male partner? Avoid exercise regimens. Minimize actions that increase scrotal heat. Sit in a hot bath twice daily to increase the scrotal heat. Use the sauna after an exercise regimen to "sweat out" the toxins.

Minimize actions that increase scrotal heat

A couple has come to the infertility clinic because they have been unable to get pregnant even though they have been trying for over a year. Diagnostic tests are planned for the woman to ascertain if ovulation is regular and whether her endometrium is adequately supported for implantation. What test would the nurse expect to have ordered for this woman? Serum progesterone Ovulation list Oocyte viability test Urine testosterone

Serum progesterone

A female patient had a diagnostic laparoscopy and is now reporting a sharp pain in her shoulder. What response is a priority by the nurse? The patient is probably having a pulmonary embolus and should be administered anticoagulation drugs immediately. The patient may be having a myocardial infarction from the stress of the procedure and the nurse should begin cardiac protocols. Some of the carbon dioxide that was used to insufflate the abdomen has likely escaped under the diaphragm and this will resolve on its own. The patient has possibly developed aspiration from stomach contents related to anesthesia.

Some of the carbon dioxide that was used to insufflate the abdomen has likely escaped under the diaphragm and this will resolve on its own.

A couple comes in and tells the nurse that they have been trying diligently to conceive for the last 3 months and are subfertile. What should the nurse explain to the couple about the definition of subfertility? Subfertility is said to exist when a pregnancy has not occurred after at least 3 months of engaging in unprotected coitus. Subfertility is said to exist when a pregnancy has not occurred after at least 6 months of engaging in unprotected coitus. Subfertility is said to exist when a pregnancy has not occurred after at least 1 year of engaging in unprotected coitus. Subfertility is said to exist when a pregnancy has not occurred after at least 2 years of engaging in unprotected coitus.

Subfertility is said to exist when a pregnancy has not occurred after at least 1 year of engaging in unprotected coitus.

A pregnant client at 10-weeks' gestation asks the nurse if the developing fetus can get infections. The nurse explains that until about 24 weeks' gestation the cytotrophoblast layer of cells protects the fetus from certain infections. The cytotrophoblast layer will protect against which of these infections? Select all that apply. Syphilis Gonorrhea Group B streptococcus HIV Toxoplasmosis

Syphilis Gonorrhea Group B streptococcus

A nurse is conducting an in-service program for a group of perinatal nurses. After teaching the group about autosomal dominant and recessive inheritance patterns, the nurse determines that the group needs additional teaching when they identify which condition as an example of an autosomal dominant disorder? neurofibromatosis achondroplasia Huntington disease Tay-Sachs disease

Tay-Sachs disease

A pregnant client is visiting the clinic and complains about the tiny, blanched, slightly raised end arterioles on her face, neck, arms, and chest. The nurse should explain that these are normal during pregnancy and referred to as which of the following? Epulis Linea nigra Striae gravidarum Telangiectasias

Telangiectasis

The nurse is teaching a pregnant teenager the importance of proper nutrition and adequate weight gain throughout the pregnancy. What is the best response when the client refuses to eat due to fear of possible weight gain? The infant will be small and could have problems. There may be little impact on the infant, but the mother can suffer complications. It will just make the baby smaller, but there are no other problems associated. The infant will be smaller but should quickly gain weight.

The infant will be small and could have problems.

In OB class, the nursing instructor is discussing the female reproductive anatomy. In order to understand the anatomy and function of the uterus, the instructor explains that there are four sections in the uterus. Which of the following are sections of the uterus? Select all that apply. Uterine isthmus Fundus Fallopian tubes Cervix Corpus

Uterine Isthmus Fundus Cervix Corpus

The nursing instructor is explaining the nursing care that is given to a client during her pregnancy. The instructor determines the session is successful when the students correctly choose which method will be used to evaluate the effectiveness of the nursing care they will provide? Ask the client to respond to a preprinted survey about nursing care. Interview the client on her first postpartum visit about the nursing care she have received. Identify the nursing diagnoses for the specific client. Verify that desired outcomes for identified goals have been met.

Verify that desired outcomes for identified goals have been met.

The nurse has just informed a client that her pregnancy test is positive and she will need further assessment to determine the complete status of the situation. Which initial emotional response does the nurse expect the client to exhibit? ambivalence introversion acceptance emotional lability

ambivalence

A pregnant female is to undergo testing to assess the karyotype of her fetus. When planning this client's care, the nurse would expect to prepare the woman for which test? Select all that apply. chorionic villi sampling amniocentesis fetoscopy quadruple marker screen fetal ultrasound

chorionic villi sampling amniocentesis

A nurse is assessing a client diagnosed with pelvic inflammatory disease (PID). Which finding would the nurse most likely assess? Select all that apply. oral temperature of 102°F (39°C) painful urination right upper quadrant pain tenderness with cervical motion negative pregnancy history clear vaginal discharge

oral temperature of 102°F (39°C) painful urination tenderness with cervical motion negative pregnancy history

When describing the menstrual cycle to a group of young women, the nurse explains that estrogen levels are highest during which phase of the endometrial cycle? menstrual proliferative secretory ischemic

proliferative

A nurse is preparing a presenstation on the menstrual cycle for a health fair. Which phase will the nurse illustrate as producing progesterone? menstrual proliferative secretory ischemic

secretory

A female patient who has been trying to conceive for 1 year comes to the fertility clinic and will be undergoing a subfertility evaluation. When discussing the components involved, which information would the nurse most likely include? Select all that apply. semen analysis of the partner ovulation monitoring hormone injections tubal patency assessment medication regimen

semen analysis of the partner ovulation monitoring tubal patency assessment

A clinic nurse is asked by a patient receiving genetic counseling about symptoms related to Turner syndrome. Which manifestation would the nurse most likely include? Select all that apply. short stature streak ovaries abnormal iris color infertility webbed neck

short stature streak ovaries infertility webbed neck

When conducting a health history for an infant child with a presumed genetic disorder, what family members with the nurse gather information on? Select all that apply. siblings parents great-grandparents grandparents in-laws

siblings parents grandparents

A nurse is teaching a class on genetic disorders for pregnant couples who have recently been told their unborn child has a genetic disorder. Which statement would be important for the nurse to include? "Genetic disorders occur at the moment an ovum and sperm fuse." "Genetic disorders occur when you expose yourself to harmful chemicals." "Genetic disorders occur because you were predestined for it." "Genetic disorders occur during proliferation of cell growth.

"Genetic disorders occur at the moment an ovum and sperm fuse."

A young couple who underwent preconceptual genetic counseling and testing have learned that they are at high risk for having a child with Down syndrome. They have decided not to have children. What would be the most appropriate response for the nurse to give? "I understand. In case you would like to discuss this further with a genetic counselor, here is the contact information for the genetic counseling center." "I appreciate your decision, but I urge you to think through this further. Having a child, even one with Down syndrome, is so rewarding." "I understand and support your decision. The risk is just not worth it." "I think you made the right decision. After all, I never had children, and I'm perfectly happy."

"I understand. In case you would like to discuss this further with a genetic counselor, here is the contact information for the genetic counseling center."

A young couple, 8 weeks pregnant with their first child, are being assessed at their first prenatal visit. Which response from the nurse is most accurate when questioned about scheduling an ultrasound to find out the gender of the baby? Today 12 weeks 16 weeks 20 weeks

16 weeks

Which change related to the vital signs is expected in pregnant women? Pulse decreases. Lung space increases. Blood pressure decreases. Temperature decreases.

BP decreases

Which of the following findings is most worrisome in Melissa, a woman in her 26th week of pregnancy? Generalized hair loss A hyperpigmented rash over the maxillary region bilaterally Nosebleeds Facial edema

Facial edema

After teaching a group of adolescent girls about female reproductive development, the nurse determines that teaching was successful when the girls state that menarche is defined as a woman's first: sexual experience full hormonal cycle menstrual period sign of breast development

menstrual period

A nurse is completing a continuing education program about the male and female reproductive organs. After reviewing the information, the nurse demonstrates understanding of the information by identifying which structures as male accessory organs? Select all that apply. testes vas deferens bulbourethral glands prostate gland penis

vas deferens bulbourethral glands prostate gland

A nurse is conducting a class geared toward changes in early pregnancy and self-care items like perineal hygiene. A woman shares that she douches at least once a day since she has "so much discharge" from her vagina. Which response by the nurse is most appropriate at this time? "Douching will definitely keep your vagina clean." "If you prepare your own douching solution, be sure to boil the water to kill bacteria." "During pregnancy, you should not douche because it can cause fluid to enter the cervix resulting in an infection." "Let's discuss this with your health care provider before you continue douching."

"During pregnancy, you should not douche because it can cause fluid to enter the cervix resulting in an infection."

After teaching a woman who has chosen the vaginal ring as her method of contraception, the nurse determines that the client needs additional teaching when she makes which statement? "It must be centered on my cervix to be effective ." "I'll compress the ring, inserting it as far back as possible." "I will insert a new ring at the same time and day of every week." "Once I remove the ring, I'll discard it."

"I will insert a new ring at the same time and day of every week"

A couple is seeking guidance for their inability to conceive a child after trying for 15 months. They are morbidly obese but state they have friends with the same weight problem who have had no difficulty conceiving. What education can the nurse provide this couple to increase their chances of success? "If you have been trying for this long without success, something must be wrong. So we will need to run a series of tests to see what that is." "Obesity may interfere with effective penetration and deposition of sperm. We will look at several factors to discover what issues you may be encountering." "Being this overweight is unhealthy for you and for any child that you may bring into the world. Before you consider conceiving, you should lose weight." "Weight has no bearing on the ability to conceive. Something else must be going on with your reproductive organs."

"Obesity may interfere with effective penetration and deposition of sperm. We will look at several factors to discover what issues you may be encountering."

The nurse is teaching a prenatal class about preparing for their expanding families. What is helpful advice from the nurse? "Expect your other children to react positively to their new brother/sister." "Your old coping methods will adequately get you through this period of adjustment." "The hormones of pregnancy may cause anxiety or depression postpartum." "Caring for your new infant is instinctual and will come naturally to you."

"The hormones of pregnancy may cause anxiety or depression postpartum."

A client is scheduled to have in vitro fertilization (IVF) in 1 week. Which statement made by the client indicates that she needs further teaching? "The primary care provider will transfer my fertilized eggs into the uterus." "The primary care provider will transfer the egg and sperm into the fallopian tube where the egg will become fertilized." "I will receive fertility drugs to stimulate ovulation." "The primary care provider will harvest eggs from my ovary."

"The primary care provider will transfer the egg and sperm into the fallopian tube where the egg will become fertilized."

A 12-year-old girl expresses concern to the nurse because she has not begun her period yet, whereas almost all of her friends have already begun menstruating. She asks when the nurse thinks she will begin her period. The nurse notes that the girl has grown 6 inches in the past year and a half and has developed breast buds and pubic hair in the past year. Which response would be best for the nurse to give the girl? "Your body is giving a lot of signs that you will probably start your period in no more than a year, which is completely normal." "It looks like you are a late bloomer! It's difficult to say when you'll get your period; some girls start as late as age 17." "Most girls have already begun their period by your age. We should run a few tests to make sure you don't have a hormone problem." "The age that you start your period is highly genetic; ask your mother when she began her period."

"Your body is giving a lot of signs that you will probably start your period in no more than a year, which is completely normal."

In teaching a fertility class, what is the most appropriate method for the nurse to explain self-ovulation monitoring that promotes conception? 14 days before the next menses is expected At the midpoint between menses cycles Immediately following the menses 14 days after the beginning of the last menses

14 days before the next menses is expected

A nurse is receiving a client from the post anesthesia unit to the recovery unit at the ambulatory surgery center. The client just had a laparoscopic tubal ligation. Which is the nurse's priority assessment? Pain Depression Bleeding Abdominal bloating

Bleeding

A client has been confirmed to be pregnant. She gives a history of two previous full-term normal pregnancies. How will the nurse classify the client's pregnancy history? G3, P0 G3, P2 G2, P1 G2, P3

G3, P2

A nurse is teaching a health class of males at a local middle school about male reproductive structures. The nurse determines that the teaching was successful when the class identifies which structure as an external male reproductive structure? Select all that apply. Testes Scrotum Penis Epididymis Vas deferens

Testes Srotum Penis

The nurse is helping a pregnant client adapt psychologically. What outcome best demonstrates that the client has successfully adapted to the second trimester? The client states that she is fully prepared for parenthood The client accepts the pregnancy The client accepts the reality and uniqueness of the baby The client finishes making a detailed plan for labor and birth

The client accepts the pregnancy

Assessment reveals that a male client is experiencing a decrease in the amount of semen produced during ejaculation. The nurse suspects that which male reproductive structure may be involved? ejaculatory ducts testes prostate scrotum

ejaculatory ducts

A pregnant client is concerned about genetic inheritance and asks the nurse which tests she can expect to undergo. What is the appropriate response by the nurse? Select all that apply. sonogram karyotyping amniocentesis umbilical blood sampling cfDNA testing

sonogram amniocentesis umbilical blood sampling cfDNA testing

A nurse is interviewing a couple at a preconception counseling session. The couple is of Greek heritage and are concerned about the possibility of their children being born with a genetic disorder. Based on the nurse's understanding of genetic disorders, the nurse would identify this couple as being at risk for which condition? β-thalassemia α-thalassemia sickle cell anemia Tay-Sachs disease

β-thalassemia

A pregnant client reports frequent urination and tells the health care provider that she has stopped drinking water during the day since she cannot take many breaks during work. Which statement by the nurse is most appropriate at this time? "Maybe it would be better to stop drinking caffeinated beverages/coffee instead of water." "Fluids are necessary so your blood volume can double, which is normal in pregnancy." "Just wait until late pregnancy when the baby's head is settled into the inlet of the pelvis." "I can write you a note to give to your supervisor if that will help relieve some stress."

"Fluids are necessary so your blood volume can double, which is normal in pregnancy."

A nurse is providing education to a client with infertility concerns. Which statement by the client indicates appropriate understanding of the Spinnbarkeit Test? "I will need to purchase a microscope and look for a fern-like pattern from a sample of my cervical mucus to indicate when I am ovulating." "I will need to purchase an over-the-counter ovulation kit that will tell me when I am currently ovulating." "Halfway in my cycle I will need to check my cervical mucus to see if I can stretch it in to long strands, indicating I am close to ovulating." "At midpoint in my menstrual cycle, I need to come to the clinic for a cervical exam looking for color changes to my cervical mucus."

"Halfway in my cycle I will need to check my cervical mucus to see if it can stretch it in to long strands, indicating I am close to ovulating"

A couple has just been notified that their unborn child carries a genetic disorder. The couple expresses concern that the insurance company will not cover the costs associated with the medical bills for the child. What is the most appropriate response by the nurse? "The insurance company may consider it a preexisting condition since you know." "There are laws in place that prohibit that from happening." "The insurance company may ask you to change policies once the baby is born." "They will charge you a higher premium every month."

"There are laws in place that prohibit that from happening."

A client is attempting to become pregnant and is using varied methods to identify ovulation. What assessment finding would most clearly suggest that the client is ovulating? The client has clear, thin cervical mucus Cervical mucus is absent The client menstruated eight days ago The client's progesterone levels are peaking

The client has clear, thin cervical mucus

Which of the following occurs in the male during the resolution phase of the sexual response cycle? The penis becomes erect due to increased pelvic congestion of blood. Involuntary spasmodic contractions occur in the penis. The male orgasm occurs usually with ejaculation of semen from the penis. The male experiences a period during which he is incapable of sexual response.

The male experiences a period during which he is incapable of sexual response.

When preparing a presentation for a group of pregnant women about fetal circulation, the nurse would point out which structure is primarily responsible for ensuring that highly oxygenated blood reaches the fetal brain? ductus venosus ductus arteriosus umbilical vein foramen ovale

foramen ovale

When describing the menstrual cycle to a group of young women, the nurse explains that estrogen levels are highest during which phase of the endometrial cycle? menstrual proliferative secretory ischemic

proliferative

The nurse is counseling a young woman who has just entered her second trimester, after an uneventful first trimester. She tells the nurse, "It still doesn't seem real. It's just hard to believe that I will really have a baby." Which future events should the nurse point out that will help the young woman come to believe it is real? Select all that apply. Feeling the baby kick Seeing an ultrasound image of the baby Giving up alcohol Receiving a positive result on a pregnancy test Taking prenatal vitamins

Feeling the baby kick Seeing an ultrasound image of the baby

A couple consults with the nurse regarding their infertility issues. The female says "I don't know what I have done so bad that God would punish me like this." What would be the most appropriate nursing diagnosis for this couple? Spiritual distress related to inability to conceive Hopelessness related to inability to conceive Powerlessness related to infertility Anticipatory grieving related to infertility

Spiritual distress related to inability to conceive

Positive signs of pregnancy are diagnostic, meaning nothing else can elicit that sign except pregnancy. What is the earliest positive sign of pregnancy? Finding of hCG in the blood Visualization of the gestational sac or fetus Finding hCG in the urine Positive home pregnancy test

Visualization of the gestational sac or fetus

The vagina is a hostile environment for sperm. What characteristic of semen protects sperm from the vaginal environment? -acidic fluid -alkaline fluid -presence of testosterone -secretions from seminiferous tubules

alkaline fluid

A pregnant client in her second trimester informs the nurse that she needs to travel by air the following week. Which precaution should the nurse instruct the client to take during the flight? Wear a padded bra. Wear low-heeled shoes. Wear support hose. Wear cotton clothes.

wear support hose

A woman in her third trimester complains to the nurse of significant back pain. The nurse questions the client carefully and records a detailed account of her back symptoms. What is the best rationale for the nurse evaluating the client's back symptoms with such care? Back pain could be a sign of bladder or kidney infection Back pain could be a sign of degenerated discs Back pain could be a result of a soft mattress Back pain could be a result of improper lifting

Back pain could be a sign of bladder or kidney infection

The birth attendant has ordered an ultrasound for a pregnant woman to determine the gestational age of the fetus. The nurse would anticipate which additional information as being obtained by this test? Select all that apply. Location of the placenta Oxygen supply to fetus Fetal heart rate related to fetal activity Gender of the fetus Fetal neural tube defects Fetal head size

Gender of the fetus Location of the placenta ' Fetal head size

The nursing instructor is teaching a class on the various hormones necessary for a successful pregnancy and birthing process. The instructor determines the session is successful when the students correctly choose which hormone as being necessary after birth to ensure growth of the newborn? Oxytocin Estrogen Progesterone Prolactin

Prolactin

The nurse is examining a woman who came to the clinic because she thinks she is pregnant. Which data collected by the nurse are presumptive signs of her pregnancy? Select all that apply. breast changes ultrasound pictures fetal heartbeat amenorrhea hydatidiform mole morning sickness

breast changes amenorrhea morning sickness

After explaining the function of the seminal vesicles to a class, the instructor determines the need for additional teaching when the students identify which action as a function of the seminal vesicles? nourishes the sperm reduces sperm motility aids sperm to reach the ovum prevents sperm destruction by female antibodies

reduces sperm motility

The nurse is explaining the various male internal reproductive sturctures in a high school health class. The nurse determines the class is successful when the class correctly chooses which structure as responsible for sperm production? seminiferous tubules vas deferens seminal vesicles spermatic cord

seminiferous tubules

A pregnant vegan reports eating lots of dark green leafy vegetables, legumes, citrus fruits, and berries. To ensure that her infant's nervous system will develop properly, what foods should the nurse recommend that she add to her diet? Milk and cheese Carrots, sweet potatoes, and mangoes Nuts, seeds, and chocolate Fortified cereals

Fortified cereals

A client is requesting information on the various available contraceptives. When explaining a vaginal spermicide, which information should the nurse prioritize? Wash with clean water and soap after intercourse. Insert the product by applicator in the vagina several hours prior to intercourse. Apply the spermicide directly to the glans penis to kill the sperm when they exit the penis. Leave the product in place for 24 hours after intercourse.

Insert the product by applicator in the vagina several hours prior to intercourse.

A nurse is talking to a group of young couples who wish to conceive. One young woman asks the nurse if any tests predict fetal abnormalities. Which of the following tests should the nurse include in the discussion? (Select all that apply.) MSAFP (maternal serum levels of alpha-fetoprotein) CVS (chorionic villi sampling) Amniocentesis CBC Lipid panel

MSAFP (maternal serum levels of alpha-fetoprotein) CVS (chorionic villi sampling) Amniocentesis

A nurse is giving a prenatal class on teratogens that have an affinity for specific body tissues. Which teratogen is accurately paired with the specific body tissue affected? Lead attacks the joints. Mercury attacks the nervous tissue. Tetracycline attacks the muscles. Thalidomide attacks the lungs.

Mercury attacks the nervous tissue.

A client calls the clinic nurse crying, saying that she just can't live with herself since she had an abortion 2 months ago. What is the priority referral for the nurse to make? Visit with the client's church pastor Find a support group for her to attend The local mental health crisis center Go have dinner with some friends

The local mental health crisis center

A woman trying to conceive has been prescribed metronidazole (Flagyl) for bacterial vaginosis. What should the nurse caution this patient while taking this medication? The medication will be continued to treat the infection. She will be monitored closely if she discovers she is pregnant. This medication is safe for use in pregnancy. This drug is the only treatment for bacterial vaginosis and must be taken as prescribed even if pregnancy is suspected. This drug can be teratogenic early in pregnancy and shouldn't be continued if a pregnancy is suspected.

This drug can be teratogenic early in pregnancy and shouldn't be continued if a pregnancy is suspected.

A nurse is teaching a class on pregnancy prevention to a group of teenagers. What should the nurse include as potential adverse affects to the child who results from an unplanned pregnancy? Select all that apply. risk for low birth weight risk of dying in the first year risk of being abused risk of not receiving sufficient resources for healthy development risk of not completing high school

risk for low birth weight risk of dying in the first year risk of being abused risk of not receiving sufficient resources for healthy development

A nurse is assessing a 26-year-old married female client for use of oral contraception. Which findings in her health history would suggest that natural family planning may be a better option? Select all that apply. abortion at age 18 menses at age 11 smokes 1 pack per day blood pressure 138/88 mm Hg migraine headaches

smokes 1 pack per day blood pressure 138/88 mm Hg migraine headaches

The school nurse is preparing a teaching session for a group of adolescent girls explaining the menstrual cycle. The nurse determines the session is successful when the young students correctly choose which fact? The average menstrual flow lasts 4 to 6 days. The average age for menarche is 11 to 13 years. Menarche is the start of puberty. Total blood loss each month averages 45 to 80 mL of blood.

the average menstrual flow lasts 4 to 6 days

Hormone levels of a woman indicate that the corpus luteum stopped functioning and releasing progesterone after 5 weeks. The nurse would recognize that which scenario is the expected outcome? The pregnancy would continue unaffected. A spontaneous abortion (miscarriage) would occur. There is a higher than normal chance of a multifetal pregnancy. She will need progesterone supplement throughout the pregnancy.

A spontaneous abortion (miscarriage) would occur.

Charlene McCoy, who has several children already, reports for a first prenatal visit. She seems preoccupied and withdrawn, and she makes consistently negative remarks about the pregnancy. Reviewing her records, you note that she is receiving a serotonin reuptake inhibitor. What should you do? Reassure her that ambivalence is normal Refer her for drug and alcohol counseling Give her printed material to read at home Alert the RN or primary care provider

Alert the RN or primary care provider

A woman who has just found out that she is pregnant tells the nurse that she takes docusate sodium (colace). The nurse identifies this drug as a category C medication. Which information best reflects the nurse's understanding of this category? Adequate studies in pregnant women have failed to show risk to the fetus Animal studies have not shown an adverse effect on the fetus, but no adequate clinical studies have been done in pregnant women. Animal studies have shown an adverse effect on the fetus, but no adequate studies have been done in humans. Pregnancy risk is unknown. Evidence of risk to the human fetus has been noted, but potential benefits of use for pregnant women may make its use acceptable despite potential risks. Studies in animals and in humans show fetal abnormalities, or adverse reaction reports indicate evidence of fetal risk. Risks involved clearly outweigh potential benefits.

Animal studies have shown an adverse effect on the fetus, but no adequate studies have been done in humans. Pregnancy risk is unknown.

A nurse is conducting an orientation at a health clinic for newly hired nurses comparing normal and abnormal assessment findings on a female physical assessment. The nurse determines the session is successful when they correctly point out which structure's secretions assist with sexual intercourse? introitus Bartholin's glands vestibule labia minora

Bartholin's glands

Which teaching strategy would be most appropriate when educating a primapara about the changes that will occur during her pregnancy? Supply the client with a popular book on pregnancy and assign readings to prepare for upcoming visits. Encourage the client to write down questions and those topics will be the information presented during clinic visits. Be selective about the information provided and include those points most relevant to this particular client. The nurse asks the client if he or she can answer any questions so that the client controls the education.

Be selective about the information provided and include those points most relevant to this particular client.

The nurse midwife is performing a pelvic examination on a client who came to her following a positive home pregnancy test. The nurse checks the woman's cervix for the probable sign of pregnancy known as Goodell's sign. Which description illustrates this alteration? The cervix looks blue or purple when examined. The lower uterine segment softens. The fundus enlarges. The cervix softens.

The cervix softens

A woman who is 8 weeks pregnant has come to the clinic for an evaluation. During the visit, the woman asks the nurse, "When will I be able to hear my baby's heartbeat?" Which response by the nurse would be most appropriate? "You need to be about 14 weeks pregnant before we can check it." "We will be able to hear it at about 10 weeks with a Doppler." "Let's try checking now to see if we can hear it." "Once you reach 12 weeks, we'll start to check it."

"We will be able to hear it at about 10 weeks with a Doppler"

A student nurse is preparing for a presentation which will illustrate the various physiologic changes in the woman's body during pregnancy. Which cardiovascular changes up through the 26th week should the student point out? Decreased pulse rate and increased blood pressure Increased pulse rate and decreased blood pressure Increased pulse rate and blood pressure No change in pulse rate or blood pressure

Increased pulse rate and decreased blood pressure

A young woman who is pregnant for the first time goes to the clinic for her first prenatal visit. During the interview, she informs the primary are provider that there is no history of genetic defects in her or her husband's family. What test will most likely be prescribed for this client as a routine screening? amniocentesis MSAFP CVS PPD

MSAFP

The school nurse is preparing a teaching session for a group of adolescent girls explaining the menstrual cycle. The nurse determines the session is successful when the young students correctly choose which fact? The average menstrual flow lasts 4 to 6 days. The average age for menarche is 11 to 13 years. Menarche is the start of puberty. Total blood loss each month averages 45 to 80 mL of blood.

The average menstrual flow lasts 4 to 6 days

A client in her second trimester of pregnancy arrives at the health care facility for a routine follow-up visit. The nurse is required to educate the client so that the client knows what to expect during her second trimester. Which information should the nurse offer? "You may feel physical discomfort as the baby inside grows." "You will experience quickening, and you will actually feel the baby." "You will be more conscious of the changes taking place in your body now." "You may have mood swings that could overwhelm your partner."

"You will experience quickening, and you will actually feel the baby."

The experienced nurse working in a fertility clinic understands the stress couples experience when they have to "schedule" sexual relations. Which suggestion would be most helpful to a couple to help prevent further stress on their relationship? Try to relax and enjoy it. Begin a new activity together. Take up new individual hobbies. Avoid talking about the cost that is involved.

Begin a new activity together

During an exam, the nurse notes that the blood pressure of a client at 22 weeks' gestation is lower, and her heart rate is 12 beats per minute higher than at her last visit. How should the nurse interpret these findings? The heart rate increase may indicate that the client is experiencing cardiac overload. The blood pressure should be higher since the cardiac volume is increased. Both findings are normal at this point of the pregnancy. Combined, both of these findings are very concerning and warrant further investigation.

Both findings are normal at this point of the pregnancy.

A 35-year-old female client who is unable to conceive is scheduled for a hysterosalpingogram to assess tubal patency. Which of the following would the nurse ensure before the client undergoes the hysterosalpingogram? Client has been administered general anesthesia Client is in second phase of menstrual cycle Client's basal body temperature is recorded Client has just completed her menses

Client has just completed her menses

The nurse is conducting a routine well-visit on a 43-year-old client who is concernd about developing breast cancer, although no family members have experienced it. What is the best advice for this client at this time? Conduct regular breast self-examination. Inspect her nipples for discharge. Ask health care provider for recommendations based on family history. Have a yearly mammogram as part of a comprehensive screening.

Conduct regular breast self-examination

After the nurse describes fetal circulation to a pregnant woman, the woman asks why her fetus has a different circulation pattern than hers. When responding to the client, the nurse integrates understanding of what information? Fetal blood is thicker than that of adults and needs different pathways. Fetal circulation carries highly oxygenated blood to vital areas first. Fetal blood has a higher oxygen saturation and circulates more slowly. Fetal heart rates are rapid and circulation time is double that of adults.

Fetal circulation carries highly oxygenated blood to vital areas first.

The nurse has assigned the nursing diagnosis Sexual dysfunction related to altered body function to a new 28-year-old male client with erectile dysfunction. Which nursing interventions should the nurse implement with this client? Select all that apply. Gather sexual history Assess for depression Refer to health care provider Explore alternative gratification methods Teach about normal changes of aging

Gather sexual history Assess for depression Refer to health care provider

When teaching a pregnant client about the physiologic changes of pregnancy, the nurse reviews the effect of pregnancy on glucose metabolism. Which underlying reason for the effect would the nurse include? Pancreatic function is affected by pregnancy. Glucose is utilized more rapidly during a pregnancy. The pregnant woman increases her dietary intake. Glucose moves through the placenta to assist the fetus.

Glucose moves through the placenta to assist the fetus.

A client is having her vital signs and weight taken and recorded at a prenatal visit. She is in her second trimester at 23 weeks' gestation. Her weight gain in the first trimester was 2 pounds and she has currently gained 14 pounds overall. What is the nurse's interpretation of this data? Her weight gain was less than expected initially but now she has gained too much weight in the second trimester. The client has displayed an ideal weight gain pattern. Her weight gain in the first trimester is less that expected but she has caught up and her weight gain is good. She needs to gain another 20 to 25 pounds by delivery to have appropriate weight gain for the pregnancy.

Her weight gain in the first trimester is less that expected but she has caught up and her weight gain is good.

A nurse practitioner (NP) is presenting information about birth control pills to a group of young women. One of the women in the group asks the NP, "When would birth control pills not be a good idea?" Which situation would the nurse include as a possible contraindication for the use of birth control pills? Select all that apply. History of thromboembolic disease Family history of cerebrovascular or cardiovascular disease 40 years or older Diabetes Smoking

History of thromboembolic disease Family history of cerebrovascular or cardiovascular disease 40 years or older Smoking

The nurse is assessing a client during a prenatal visit. While listening to fetal heart tones, the client tells the nurse, "I"ve been itching pretty much all over my body." The rest of teh client's assessment is unremarkable. The nurse suspects that the client's complaint is the result of which body change? Rising levels of progesterone Increasing immune response Increasing reabsorption of bilirubin into the maternal bloodstream Increasing levels of estrogen

Increasing reabsorption of bilirubin into the maternal bloodstream

A couple has just learned that their unborn son has a chromosome disorder that results in an extra X chromosome. The primary care provider explains that secondary sex characteristics will not develop in this child at puberty and that his testes will remain small and produce ineffective sperm. The nurse recognizes that this child likely has: Klinefelter syndrome. Turner syndrome. fragile X syndrome. Down syndrome.

Klinefelter syndrome.

An expectant mother asks the nurse which of her antibodies from previous immunizations would be passed to her fetus during pregnancy. Which response by the nurse would be correct? Measles antibodies will cross the placenta to protect the fetus. If the mother was immunized within the last year for varicella, the immunity will pass to the fetus. Since the flu vaccine is passed easily, the fetus will not need a flu shot for 2 years. Cytomegalovirus is one of the first passive antibodies to cross the placenta.

Measles antibodies will cross the placenta to protect the fetus.

A client is at 20 weeks' gestation and is scheduled for a fetal survey with ultrasound. What will the ultrasound reveal about the placenta? (Select all that apply.) Placement of the placenta Functional grade of the placenta Size of the placenta Down syndrome (Trisomy 21) in the fetus Maternal human immunodeficiency virus (HIV) status

Placement of the placenta Functional grade of the placenta Size of the placenta

The nurse is counseling a couple who suspect that they could bear a child with a genetic abnormality. What would be most important for the nurse to do when working with this family? Gather information for three generations. Inform the family of the need for information. Maintain the confidentiality of the information. Present the information in a factual, nondirective manner.

Present the information in a factual, nondirective manner.

What effect does progesterone have on normal gallbladder function? It has no effect on the gallbladder. The gallbladder will hypertrophy. Progesterone interferes with gallbladder contraction, leading to stasis of bile. Bile will be produced at a more rapid rate due to the progesterone.

Progesterone interferes with gallbladder contraction, leading to stasis of bile.

A woman is preparing for intauterine insemination because she has been unable to conceive with her husband for more than 18 months. The woman demonstrates understanding of the procedure when she identifies which action as important in preparing for this procedure? Select all that apply. Recording basal body temperature (BBT). Assessing cervical mucus. Using an ovulation predictor kit. Douching prior to the procedure. Taking progesterone.

Recording basal body temperature (BBT). Assessing cervical mucus. Using an ovulation predictor kit.

A newly married female is talking to the nurse about her desire to get pregnant soon. Which suggestion would the nurse make to assist this client? Sexual intercourse should occur between Day 13 and 15 of her menstrual cycle. It is recommended to check her mucus and have intercourse when the mucus is opaque and scant. If she has irregular menstrual cycles, that should not interfere with her conceiving as long as she has intercourse on the same days every month. If ovulation occurs on Day 14 of her menstrual cycle, she should have intercourse one week later on Day 21.

Sexual intercourse should occur between Day 13 and 15 of her menstrual cycle.

A young couple is having difficulty getting pregnant. The nurse is preparing the couple for the initial tests to determiine their fertility. When asked by the couple why they need to start with a sperm analysis, what will be the nurse's best response? Asking the male to undergo diagnostic procedures first is the best way to assess interest in treatment of infertility. The man is found to be fertile in over 75 percent of couples with infertility problems. Male fertility testing is time-consuming and therefore should be initiated early. Sperm analysis is one of the easiest tests to complete.

Sperm analysis is one of the easiest tests to complete.

After teaching a class about conception, the nursing instructor asks the attendees to explain the process of fertilization. Which statement indicates the students need additional teaching? Select all that apply. The ovum is receptive to conception 12 to 23 hours after ovulation. Sperm are viable for only about 48 hours after ejaculation. Sperm are able to fertilize the ovum from 3 days before until 2 days after ovulation. Sperm are able to fertilize the ovum up to 12 hours after ejaculation, and the ovum remains fertile for at least 24 hours after ovulation. This activity usually occurs in the ampulla of the fallopian tube.

Sperm are able to fertilize the ovum up to 12 hours after ejaculation, and the ovum remains fertile for at least 24 hours after ovulation. ' Sperm are viable for only about 48 hours after ejaculation.

During the initial assessent of a 22-year-old pregnant client, the nurse learns that the client usually smokes 2 packs of cigarettes per day. The nurse is planning an education session about lifestyle changes during pregnancy. Which goal would be the most realistic and individualized for this client during this initial clinic visit? The client stops smoking immediately for the health of the fetus. The client reduces her smoking by 50 percent by the next clinic visit. The client throws the cigarettes in the trash immediately. The client does some research on the harmful affects of cigarette smoking on the baby.

The client reduces her smoking by 50 percent by the next clinic visit.

After teaching a class on the various structures formed by the embryonic membranes, the nurse determines that the teaching was successful when the class identifies which structure as being formed by the mesoderm? lungs bones ears stomach

bones

Which statement should be incorporated into the teaching plan developed to present instruction about the female menstrual cycle to nursing students? Menses ensues when the levels of estrogen and progesterone fall. The follicular phase is dominated by progesterone and testosterone. Progesterone influences the growth of endometrial lining. Estrogen is the dominant hormone during the luteal phase.

menses ensues when the levels of estrogen and progesterone fall

A clinic nurse is interviewing a young client during a subfertility work up. When the client asks the nurse what causes infertility, the nurse informs the client that the problem can rest with the man, the woman, or both. What does the nurse tell the client are common problem areas related to the woman? Select all that apply. ovulation tubal transport diabetes impaired implantation lack of exercise

ovulation tubal transport impaired implantation

The nurse is developing a presentation for a group of young adult women about premenstrual syndrome. Which treatment options should the nurse point out as most appropriate? Select all that apply. antipsychotic medications reduction of caffeine intake vitamin and mineral supplements decrease in water intake NSAIDs

reduction of caffeine intake vitamin and mineral supplements NSAIDs

A sex education class is being conducted at a local women's center. The attendees ask the professional conducting the class about why they experience sensations of increased warmth during sex and redness of the neck and chest. Which information would the professional provide to explain this experience? Select all that apply. A person's heart rate increases during sex, making them warmer. The blood pressure will increase, which can cause a feeling of warmth. The color change is coincidental and not a part of the sexual response. Vessels in the pelvis are receiving more blood and can cause a warm sensation. Respirations will decrease during sex, which allows the body to warm up faster.

A person's heart rate increases during sex, making them warmer The blood pressure will increase, which cause a feeling of warmth Vessels in the pelvis are receiving more blood and can cause a warm sensation

A young woman is having trouble conceiving and has come in to the office for fertility testing. She states that she has irregular and unpredictable menstrual cycles. On blood testing, it is found that her ovaries are producing excess testosterone, which is lowering her follicle-stimulating hormone (FSH) and luteinizing hormone (LH) levels. Which of the following is the most likely cause of her subfertility? Anovulation Endometriosis Tubal transport problems Stenotic cervical os

Anovulation

A client at 32 weeks' gestation telephones the health care provider's office asking if omeprazole 20mg daily is safe to take as she is experiencing heartburn. When reviewing the over-the-counter medication, the nurse notes a Pregnancy category C listed. Which would be the nurse's first action? Respond that since it is over-the-counter, it is safe. Encourage the client to eat less spicy foods and avoid all medications. Offer to speak with the health care provider and then return her call. Instruct the client that due to the pregnancy category, this medication is not taken.

Offer to speak with the health care provider and then return her call.

A client is at 20 weeks' gestation and is scheduled for a fetal survey with ultrasound. The nurse teaches the client about the test The nurse determines that the teaching was successful when the client identifies that the ultrasound will reveal which information about the placenta? Select all that apply. Placement of the placenta Functional grade of the placenta Size of the placenta Down syndrome (Trisomy 21) in the fetus Maternal human immunodeficiency virus (HIV) status

Placement of the placenta

A woman trying to conceive has been diagnosed with a luteal phase defect. Which medication would the nurse most likely anticipate as being prescribed for this woman? Clomiphene citrate (Clomid) Progesterone vaginal suppositories Metronidazole (Flagyl) Conjugated estrogen (Premarin)

Progesterone vaginal suppositories

The nursing instructor is teaching students about normal changes of pregnancy. The instructor talks about diastasis recti. What is the instructor presenting? Separation of the muscles of the abdominal wall Raising of the uterus into the abdomen Relaxation of the kidneys Movement of the bladder to the rear of the pelvis behind the uterus

Separation of the muscles of the abdominal wall

The nurse has been working with a subfertile couple through the process of determining their best plan of action to conceive a child. Which action should the nurse prioritize to provide emotional support to this couple? Speak only with both partners present so they will feel more comfortable. Speak separately with each partner so both will feel more free to talk. Speak both separately and together with both partners. Only speak with the woman, who is the client.

Speak both separately and together with both partners

A woman arrives in the emergency department in active labor. She has not received any prenatal care and reports spending a lot of time in a hot tub to relax. The nurse caring for this woman should prepare for which possible birth defect related to the fetus being exposed to prolonged hyperthermia in a hot tub? Select all that apply. white, frothy bubbles in baby's mouth coughing or choking during feedings intestines located outside the abdominal wall large opening in the baby's face and mouth swallowing problems with liquids coming out the baby's nose

white, frothy bubbles in baby's mouth coughing or choking during feedings intestines located outside the abdominal wall

A woman at 15 weeks' gestation who works at a daycare center thinks she may have just been exposed to rubella at work. The client asks how this may affect her fetus. What is the best response the nurse can give? "By the end of the eighth week all of the organ systems and major structures are present, so exposure to any teratogen can lead to birth defects. More assessments are needed." "Your health care provider will let you know if there are any problems with your baby." "We will have to see what gestational age your baby was at exposure." "We will need to perform some additional tests."

"By the end of the eighth week all of the organ systems and major structures are present, so exposure to any teratogen can lead to birth defects. More assessments are needed."

The health care provider is concerned about a client's fetus having appropriate blood flow through the fetal vessels. Which diagnostic test does the nurse anticipate to confirm a diagnosis? Transabdominal ultrasound Doppler study Amniocentesis Maternal serum alpha-fetoprotein screening

Doppler study

A woman has recently given birth to an infant with Down syndrome. She and her husband knew ahead of time, based on genetic testing, that the child would have this disorder. They are now asking the nurse for input on how this will affect their parenting. What information should the nurse give to the parents? Select all that apply. Feed the child slowly, as the enlarged tongue may interfere with swallowing. Enroll the child in early educational and play programs so he can develop to his full capacity. Use good hand washing technique, as he will be prone to infections. When older, have an x-ray of his neck taken before he engages in strenuous activity, as his neck may not be fully stable. Expect maladaptive behaviors such as hyperactivity, aggression, and autism before puberty. He will likely develop gynecomastia (increased breast size) and have increased risk for male breast cancer.

Feed the child slowly, as the enlarged tongue may interfere with swallowing. Enroll the child in early educational and play programs so he can develop to his full capacity. Use good hand washing technique, as he will be prone to infections. When older, have an x-ray of his neck taken before he engages in strenuous activity, as his neck may not be fully stable.

A client at 29-weeks gestation tells the nurse she is experiencing aches in her hips and joints. What would the nurse do next? Have the primary health care provider see the client next Tell the client these are normal findings during pregnancy Ask the client if there is a family history of arthritis Document these findings in the client's chart

Tell the client these are normal findings during pregnancy

The nurse is explaining the phases of the uterine cycle to a client. What action during the proliferative phase of the uterine cycle should the nurse include in the teaching session with the client? While the ovarian follicles are producing increased amounts of estrogen, the endometrium prepares for possible fertilization with pronounced growth. If fertilization does not occur, the corpus luteum degenerates, and hormonal levels fall. Withdrawal of hormones causes the endometrial cells to change, and menstruation begins. An ovum matures and is released.

While the ovarian follicles are producing increased amounts of estrogen, the endometrium prepares for possible fertilization with pronounced growth


Related study sets

Economics topic 3 lessons 4-6 test

View Set

Principles of Management- Exam #1

View Set

ELECTRICITY, ELECTROMAGNETISM, MOTORS, GENERATORS, TRANSFORMERS, RECTIFICATION RAD 1130

View Set

bio 112 kahoot quiz questions (exam 1)

View Set

MKTG351 Final - Alternative Selection

View Set